You are on page 1of 106

16.

1 SOLUTIONS 1253
CHAPTER SIXTEEN
Solutions for Section 16.1
Exercises
1. Mark the values of the function on the plane, as shown in Figure 16.1, so that you can guess respectively at the smallest
and largest values the function takes on each small rectangle.
Lower sum =

f(xi, yi)xy
= 4xy + 6xy + 3xy + 4xy
= 17xy
= 17(0.1)(0.2) = 0.34.
Upper sum =

f(xi, yi)xy
= 7xy + 10xy + 6xy + 8xy
= 31xy
= 31(0.1)(0.2) = 0.62.
1.0 1.1 1.2
2.0
2.2
2.4
x

y
5
4
3
7
6
5
10
8
4
E '
x = 0.1
T
c
y = 0.2
Figure 16.1
2. Using x = 3 and y = 0.5:
Lower estimate = (4 + 5 + 3 + 4)xy = 16 3 0.5 = 24,
Upper estimate = (7 + 10 + 5 + 7)xy = 29 3 0.5 = 43.5.
3. There are nine squares. Using the largest value of g in each square for the overestimate and the smallest value for the
underestimate, we have
Overestimate (2 + 2.8 + 3.5 + 2.8 + 4 + 4.9 + 3.4 + 4.9 + 6)xy = 34.3 2 2 = 137.2.
Underestimate (0 + 0.8 + 0.9 + 0.8 + 2 + 2.8 + 0.9 + 2.8 + 4)xy = 15 2 2 = 60.
4. In the subrectangle in the top left in Figure 16.5, it appears that f(x, y) has a maximum value of about 9. In the subrect-
angle in the top middle, f(x, y) has a maximum value of 10. Continuing in this way, and multiplying by x and y, we
have
Overestimate = (9 + 10 + 12 + 7 + 8 + 10 + 5 + 7 + 8)(10)(5) = 3800.
Similarly, we nd
Underestimate = (7 + 7 + 8 + 4 + 5 + 7 + 1 + 3 + 6)(10)(5) = 2400.
Thus, we expect that
2400
_
R
f(x, y)dA 3800.
1254 Chapter Sixteen /SOLUTIONS
5. (a) If we take the partition of R consisting of just R itself, we get
Lower bound for integral = minRf AR = 0 (4 0)(4 0) = 0.
Similarly, we get
Upper bound for integral = maxRf AR = 4 (4 0)(4 0) = 64.
(b) The estimates asked for are just the upper and lower sums. We partition R into subrectangles R
(a,b)
of width 2 and
height 2, where (a, b) is the lower-left corner of R
(a,b)
. The subrectangles are then R
(0,0)
, R
(2,0)
, R
(0,2)
, and R
(2,2)
,
as in Figure 16.2. Then we nd the lower sum
Lower sum =

(a,b)
AR
(a,b)
min
R
(a,b)
f =

(a,b)
4 (Min of f on R
(a,b)
)
= 4

(a,b)
(Min of f on R
(a,b)
)
= 4(f(0, 0) +f(2, 0) +f(0, 2) +f(2, 2))
= 4(

0 0 +

2 0 +

0 2 +

2 2)
= 8.
Similarly, the upper sum is
Upper sum = 4

(a,b)
(Max of f on R
(a,b)
)
= 4(f(2, 2) +f(4, 2) +f(2, 4) +f(4, 4))
= 4(

2 2 +

4 2 +

2 4 +

4 4)
= 24 + 16

2 46.63.
The upper sum is an overestimate and the lower sum is an underestimate, so we can get a better estimate by averaging
them to get 16 + 8

2 27.3.
(0, 0)
(0, 4)
(4, 0)
(4, 4)
Figure 16.2
6. (a) We rst nd an over- and underestimate of the integral, using four subrectangles. On the rst subrectangle (0 x
3, 0 y 4), the function f(x, y) appears to have a maximum of 100 and a minimum of 79. Continuing in this
way, and using the fact that x = 3 and y = 4, we have
Overestimate = (100 + 90 + 85 + 79)(3)(4) = 4248,
and
Underestimate = (79 + 68 + 61 + 55)(3)(4) = 3156.
A better estimate of the integral is the average of the overestimate and the underestimate:
Better estimate =
4248 + 3156
2
= 3702.
(b) The average value of f(x, y) on this region is the value of the integral divided by the area of the region. Since the
area of R is (6)(8) = 48, we approximate
Average value =
1
Area
_
R
f(x, y)dA
1
48
3702 = 77.125.
We see in the table that the values of f(x, y) on this region vary between 55 and 100, so an average value of 77.125
is reasonable.
16.1 SOLUTIONS 1255
7. Partition R into subrectangles with the lines x = 0, x = 0.5, x = 1, x = 1.5, and x = 2 and the lines y = 0, y = 1,
y = 2, y = 3, and y = 4. Then we have 16 subrectangles, each of which we denote R
(a,b)
, where (a, b) is the location of
the lower-left corner of the subrectangle.
We want to nd a lower bound and an upper bound for the volume above each subrectangle. The lower bound for the
volume of R
(a,b)
is
0.5(Min of f on R
(a,b)
)
because the area of R
(a,b)
is 0.5 1 = 0.5. The function f(x, y) = 2 + xy increases with both x and y over the whole
region R, as shown in Figure 16.3. Thus,
Min of f on R
(a,b)
= f(a, b) = 2 +ab,
because the minimum on each subrectangle is at the corner closest to the origin.
x
y
z
Figure 16.3
(a, b)
'
R(a, b)
y
2
4
x
(a, b)
Figure 16.4
Similarly,
Max of f on R
(a,b)
= f(a + 0.5, b + 1) = 2 + (a + 0.5)(b + 1).
So we have
Lower sum =

(a,b)
0.5(2 +ab) = 0.5

(a,b)
(2 +ab)
= 16 + 0.5

(a,b)
ab
Since a = 0, 0.5, 1, 1.5 and b = 0, 1, 2, 3, expanding this sum gives
Lower sum = 16 + 0.5 ( 0 0 + 0 1 + 0 2 + 0 3
+ 0.5 0 + 0.5 1 + 0.5 2 + 0.5 3
+ 1 0 + 1 1 + 1 2 + 1 3
+ 1.5 0 + 1.5 1 + 1.5 2 + 1.5 3)
= 25.
Similarly, we can compute the upper sum:
Upper sum =

(a,b)
0.5(2 + (a + 0.5)(b + 1)) = 0.5

(a,b)
(2 + (a + 0.5)(b + 1))
= 16 + 0.5

(a,b)
(a + 0.5)(b + 1)
= 41.
8. Since f(x, y) is measured in micrograms per square meter, and we are integrating over an area measured in square
meters, the units of the integral are micrograms. The integral represents the total quantity of pollution, in micrograms, in
the region R.
1256 Chapter Sixteen /SOLUTIONS
Problems
9. The function being integrated is f(x, y) = 1, which is positive everywhere. Thus, its integral over any region is positive.
10. The function being integrated is f(x, y) = 1, which is positive everywhere. Thus, its integral over any region is positive.
11. The function being integrated is f(x, y) = 5x. Since x > 0 in R, f is positive in R and thus the integral is positive.
12. The function being integrated is f(x, y) = 5x, which is an odd function in x. Since B is symmetric with respect to x, the
contributions to the integral cancel out, as f(x, y) = f(x, y). Thus, the integral is zero.
13. The region D is symmetric both with respect to x and y axes. The function being integrated is f(x, y) = 5x, which is an
odd function in x. Since D is symmetric with respect to x, the contributions to the integral cancel out. Thus, the integral
of the function over the region D is zero.
14. The function being integrated, f(x, y) = y
3
+ y
5
, is an odd function in y while D is symmetric with respect to y. Then,
by symmetry, the positive and negative contributions of f will cancel out and thus its integral is zero.
15. In a region such as B in which y < 0, the quantity y
3
+y
5
is less than zero. Thus, its integral is negative.
16. The region R is symmetric with respect to y and the integrand is an odd function in y, so the integral over R is zero.
17. The function being integrated, f(x, y) = y y
3
is always negative in the region B since in that region 1 < y < 0 and
|y
3
| < |y|. Thus, the integral is negative.
18. The function being integrated, f(x, y) = y y
3
, is an odd function in y while D is symmetric with respect to y. By
symmetry, the integral is zero.
19. The region D is symmetric both with respect to x and y axes. The function being integrated is odd with respect to y in the
region D. Thus, its integral is zero.
20. Since D is a disk of radius 1, in the region D, we have |y| < 1. Thus, /2 < y < /2. Thus, cos y is always positive
in the region D and thus its integral is positive.
21. The function f(x, y) = e
x
is positive for any value of x. Thus, its integral is always positive for any region, such as D,
with nonzero area.
22. The region D is symmetric both with respect to x and y axes. Looking at the contributions to the integral of the function
f(x, y) = xe
x
, we can see that any contribution made by the point (x, y), where x > 0, is greater than the corresponding
contribution made by (x, y), since e
x
> 1 > e
x
for x > 0. Thus, the integral of f in the region D is positive.
23. The region D is symmetric both with respect to x and y axes. The function f(x, y) = xy
2
is odd with respect to x, and
thus the contributions to the integral from (x, y) and (x, y) cancel. Thus the integral is zero in the region D.
24. The function f(x, y) is odd with respect to x, and thus the integral is zero in region B, which is symmetric with respect
to x.
25. The question is asking which graph has more volume under it, and from inspection, it appears that it would be the graph
for the mosquitos.
26. The total area of the square Ris (1.5)(1.5) = 2.25. See Figure 16.5. On a disk of radius 0.5 the function has a value of 3
or more, giving a total contribution to the integral of at least (3)(0.5
2
) 2.3. On less than half of the rest of the square
the function has a value between 2 and 0, giving a contribution to the integral of between (1/22.25)(2) = 2.25 and
0. Since the positive contribution to the integral is therefore greater in magnitude than the negative contribution,
_
R
f dA
is positive.
1.0 0.5 0 0.5 1.0 1.5 2.0
1.0
0.5
0
0.5
1.0
1.5
2.0
x
y

1
0
1
2
2
3
4
Figure 16.5
16.1 SOLUTIONS 1257
27. Lets break up the room into 25 sections, each of which is 1 meter by 1 meter and has area A = 1.
We shall begin our sum as an upper estimate starting with the lower left corner of the room and continue across the
bottom and moving upward using the highest temperature, Ti, in each case. So the upper Riemann sum becomes

25
i=1
TiA = T1A+T2A+T3A+ +T25A
= A(T1 +T2 +T3 + +T25)
= (1)( 31 +29 + 28 + 27 + 27+
29 +28 + 27 + 27 + 26+
27 +27 + 26 + 26 + 26+
26 +26 + 25 + 25 + 25+
25 +24 + 24 + 24 + 24)
= (1)(659) = 659.
In the same way, the lower Riemann sum is formed by taking the lowest temperature, ti, in each case:

25
i=1
tiA = t1A+t2A+t3A+ +t25A
= A(t1 +t2 +t3 + +t25)
= (1)( 27 +27 + 26 + 26 + 25+
26 +26 + 25 + 25 + 25+
25 +24 + 24 + 24 + 24+
24 +23 + 23 + 23 + 23+
23 +21 + 20 + 21 + 22)
= (1)(602) = 602.
So, averaging the upper and lower sums we get: 630.5.
To compute the average temperature, we divide by the area of the room, giving
Average temperature =
630.5
(5)(5)
25.2

C.
Alternatively we can use the temperature at the central point of each section A. Then the sum becomes

25
i=1
T

i
A = A

25
i=1
T

i
= (1)( 29 +28 + 27 + 26.5 + 26+
27 +27 + 26 + 26 + 25.5+
26 +25.5 + 25 + 25 + 25+
25 +24 + 24 + 24 + 24+
24 +23 + 22 + 22.5 + 23)
= (1)(630) = 630.
Then we get
Average temperature =

25
i=1
T

i
A
Area
=
630
(5)(5)
25.2

C.
28. We use four subrectangles to nd an overestimate and underestimate of the integral:
Overestimate = (15 + 9 + 9 + 5)(4)(3) = 456,
Underestimate = (5 + 2 + 3 + 1)(4)(3) = 132.
A better estimate of the integral is the average of the two:
_
R
f(x, y)dA
456 + 132
2
= 294.
The units of the integral are milligrams, and the integral represents the total number of mg of mosquito larvae in this 8
meter by 6 meter section of swamp.
1258 Chapter Sixteen /SOLUTIONS
29. Let R be the region 0 x 60, 0 y 8. Then
Volume =
_
R
w(x, y) dA
Lower estimate: 102(1+4+8+10+10+8+0+3+4+6+6+4+0+1+2+3+3+2+0+0+1+1+1+1) = 1580.
Upper estimate:
10 2(8+13+16+17+17+16+4+8+10+11+11+10+3+4+6+7+7+6+1+2+3+4+4+3) = 3820.
The average of the two estimates is 2700 cubic feet.
30. (a) On the xy-plane, z = 0, so the equation of the edge of the base is x
2
+y
2
= 15, a circle of radius

15.
(b) The area of the base is (

15)
2
= 15 meters
2
.
(c) The cross-section at z = 10 has equation x
2
+y
2
= 5, a circle of radius

5.
(d) The area of the cross-section is (

5)
2
= 5 meters
2
.
(e) At height z, the cross-section is the circle
x
2
+y
2
= 15 z.
This is a circle of radius

15 z, so
A(z) = (

15 z)
2
= (15 z) meters
2
.
(f) The approximate volume of the slice is A(z)z meters
3
.
(g) We have
Volume of pile =
_
15
0
(15 z) dz =
_
15z
z
2
2
_

15
0
=
225
2
meters
3
.
31. Let
f(x, y) =
1
1 +a1x
2
+y
2
and let
g(x, y) =
1
1 +a2x
2
+y
2
where 0 < a1 < a2. For all points (x, y) we have f(x, y) g(x, y) > 0, so in Riemann sum approximations for
_
R
f dA and
_
R
g dA using the same subdivision of R for both integrals, we have

f(xi, yj)xy

g(xi, yj)xy.
It follows that
_
R
f dA
_
R
g dA, and so increasing the value of a decreases the value of the integral
_
R
1
1+ax
2
+y
2
dA.
32. Take a Riemann sum approximation to
_
R
fdA

i,j
f(xi, yi)A.
Then, using the fact that |a +b| |a| +|b| repeatedly, we have:

_
R
fdA

i,j
f(xi, yi)A|

|f(xi, yi)A|.
Now |f(xi, yj)A| = |f(xi, yj|A since A is non-negative, so

_
R
fdA

i,j
|f(xi, yi)A| =

i,j
|f(xi, yj)|A.
But the last expression on the right is a Riemann sum approximation to the integral
_
R
|f|dA, so we have

_
R
fdA

i,j
f(xi, yj)A

i,j
|f(xi,j )|A
_
R
|f|dA.
Thus,

_
R
fdA

_
R
|f|dA.
16.2 SOLUTIONS 1259
Solutions for Section 16.2
Exercises
1. See Figure 16.6.

x
y
Figure 16.6
1
2
1
1
x
y
Figure 16.7
2. See Figure 16.7.
3. See Figure 16.8.
1
1
x
y
Figure 16.8
2 4
1
2
x
y
Figure 16.9
4. See Figure 16.9.
5. We evaluate the inside integral rst:
_
4
0
(4x + 3y) dx = (2x
2
+ 3yx)

4
0
= 32 + 12y.
Therefore, we have
_
3
0
_
4
0
(4x + 3y) dxdy =
_
3
0
(32 + 12y) dy = (32y + 6y
2
)

3
0
= 150.
6. We evaluate the inside integral rst:
_
3
0
(x
2
+y
2
) dy =
_
x
2
y +
y
3
3
_

y=3
y=0
= 3x
2
+ 9.
Therefore, we have
_
2
0
_
3
0
(x
2
+y
2
) dydx =
_
2
0
(3x
2
+ 9) dx = (x
3
+ 9x)

2
0
= 26.
1260 Chapter Sixteen /SOLUTIONS
7. We evaluate the inside integral rst:
_
2
0
(6xy) dy = (3xy
2
)

2
0
= 12x.
Therefore, we have
_
3
0
_
2
0
(6xy) dydx =
_
3
0
(12x) dx = (6x
2
)

3
0
= 54.
8. We evaluate the inside integral rst:
_
2
0
(x
2
y) dy =
_
x
2
y
2
2
_

y=2
y=0
= 2x
2
.
Therefore, we have
_
1
0
_
2
0
(x
2
y) dydx =
_
1
0
(2x
2
) dx =
_
2x
3
3
_

1
0
=
2
3
.
9. Calculating the inner integral rst, we have
_
1
0
_
1
0
ye
xy
dxdy =
_
1
0
_
e
xy

1
0
_
dy =
_
1
0
_
e
y
e
0
_
dy =
_
1
0
(e
y
1) dy = (e
y
y)

1
0
= e
1
1(e
0
0) = e2.
10. Calculating the inner integral rst, we have
_
3
0
_
y
0
sin xdxdy =
_
3
0
_
cos x

y
0
_
dy =
_
3
0
(cos y + 1) dy = (sin y +y)

3
0
= sin 3 + 3.
11. Calculating the inner integral rst, we have
_
2
0
_

y
0
2xcos(x
2
) dxdy =
_
2
0
_
sin(x
2
)

y
0
_
dy =
_
2
0
(sin y sin 0) dy =
_
2
0
sin y dy
= cos y

2
0
= cos 2 + cos 0 = 1 cos 2.
12.
_
4
1
_
2
1
f dy dx or
_
2
1
_
4
1
f dxdy
13. This region lies between x = 0 and x = 4 and between the lines y = 3x and y = 12, and so the iterated integral is
_
4
0
_
12
3x
f(x, y) dydx.
Alternatively, we could have set up the integral as follows:
_
12
0
_
y/3
0
f(x, y) dxdy.
14. The line connecting (1, 1) and (3, 2) is
3x + 4y = 1
or
y =
1 3x
4
So the integral becomes
_
3
1
_
(13x)/4
2
f dy dx or
_
1
2
_
(14y)/3
1
f dxdy
16.2 SOLUTIONS 1261
15. The line on the left (through points (0, 0) and (3, 6)) is the line y = 2x; the line on the right (through points (3, 6) and
(5, 0)) is the line y = 3x + 15. See Figure 16.10. One way to set up this iterated integral is:
_
6
0
_
(15y)/3
y/2
f(x, y) dxdy.
The other option for setting up this integral requires two separate integrals, as follows:
_
3
0
_
2x
0
f(x, y) dydx +
_
5
3
_
3x+15
0
f(x, y) dydx.
R
y = 2x
y = 3x + 15
x
y
Figure 16.10
16. Two of the sides of the triangle have equations x =
y 1
2
and x =
y 5
2
. So the integral is
_
3
1
_

1
2
(y5)
1
2
(y1)
f dxdy
17. The line connecting (1, 0) and (4, 1) is
y =
1
3
(x 1)
So the integral is
_
4
1
_
2
(x1)/3
f dy dx
18.
_
3
1
_
4
0
e
x+y
dxdy =
_
3
1
e
x
e
y

4
0
dx =
_
3
1
e
x
(e
4
1) dx = (e
4
1)(e
2
1)e. See Figure 16.11.
1 3
y = 4
x
y
Figure 16.11
2 2
x
y
y = x
Figure 16.12
19.
_
2
0
_
x
0
e
x
2
dydx =
_
2
0
e
x
2
y

x
0
dx =
_
2
0
xe
x
2
dx =
1
2
e
x
2

2
0
=
1
2
(e
4
1). See Figure 16.12.
1262 Chapter Sixteen /SOLUTIONS
20.
_
5
1
_
2x
x
sin xdy dx =
_
5
1
sin x y

2x
x
dx
=
_
5
1
sin x xdx
= (sin x xcos x)

5
1
= (sin 5 5 cos 5) (sin 1 cos 1) 2.68.
See Figure 16.13.
1 5
x
y
y = x
y = 2x
Figure 16.13
1 2 4
1
4
x
y
y = 4
y = x
x =

y
Figure 16.14
21.
_
4
1
_
y

y
x
2
y
3
dxdy =
_
4
1
y
3
x
3
3

y
dy
=
1
3
_
4
1
(y
6
y
9
2
) dy
=
1
3
_
y
7
7

y
11/2
11/2
_

4
1
=
1
3
__
4
7
7

4
11/2
2
11
_

_
1
7

2
11
_
_
656.082
See Figure 16.14.
22. The region of integration ranges from x = 0 to x = 3 and from y = 0 to y = 2x, as shown in Figure 16.15. To evaluate
the integral, we evaluate the inside integral rst:
_
2x
0
(x
2
+y
2
) dy =
_
x
2
y +
y
3
3
_

y=2x
y=0
= x
2
(2x) +
(2x)
3
3
= 2x
3
+
8x
3
3
=
14
3
x
3
.
Therefore, we have
_
3
0
_
2x
0
(x
2
+y
2
) dydx =
_
3
0
_
14
3
x
3
_
dx =
_
14
12
x
4
_

3
0
= 94.5.
3
6
y = 2x
x
y
Figure 16.15
3 2 3
3
3
x
y

x
2
+ y
2
= 9
Figure 16.16
16.2 SOLUTIONS 1263
23. See Figure 16.16.
_
0
2
_
0

9x
2
2xy dydx =
_
0
2
xy
2

9x
2
dx
=
_
0
2
x(9 x
2
) dx
=
_
0
2
(x
3
9x) dx
=
_
x
4
4

9
2
x
2
_

0
2
= 4 + 18 = 14
24.
_
R

x +y dA =
_
2
0
_
1
0

x +y dxdy
=
_
2
0
2
3
(x +y)
3
2

1
0
dy
=
2
3
_
2
0
((1 +y)
3
2
y
3
2
) dy
=
2
3

2
5
[(1 +y)
5
2
y
5
2
]

2
0
=
4
15
((3
5
2
2
5
2
) (1 0))
=
4
15
(9

3 4

2 1) = 2.38176
25. In the other order, the integral is
_
1
0
_
2
0

x +y dy dx.
First we keep x xed and calculate the inside integral with respect to y:
_
2
0

x +y dy =
2
3
(x +y)
3/2

y=2
y=0
=
2
3
_
(x + 2)
3/2
x
3/2

.
Then the outside integral becomes
_
1
0
2
3
_
(x + 2)
3/2
x
3/2

dx =
2
3
_
2
5
(x + 2)
5/2

2
5
x
5/2
_

1
0
=
2
3

2
5
_
3
5/2
1 2
5/2

= 2.38176
Note that the answer is the same as the one we got in Exercise 24.
26.
_
R
(5x
2
+ 1) sin 3y dA =
_
1
1
_
/3
0
(5x
2
+ 1) sin 3y dy dx
=
_
1
1
(5x
2
+ 1)
_

1
3
cos 3y

/3
0
_
dx
1264 Chapter Sixteen /SOLUTIONS
=
2
3
_
1
1
(5x
2
+ 1) dx
=
2
3
(
5
3
x
3
+x)

1
1
=
2
3
(
10
3
+ 2) =
32
9
27. The region of integration, R, is shown in Figure 16.17.
Integrating rst over y, as shown in the diagram, we obtain
_
R
xy dA =
_
1
0
__
1x
0
xy dy
_
dx =
_
1
0
xy
2
2

1x
0
dx =
_
1
0
1
2
x(1 x)
2
dx
Now integrating with respect to x gives
_
R
xy dA =
_
1
4
x
2

1
3
x
3
+
1
8
x
4
_

1
0
=
1
24
.
1
1
y = 1 x
x
y
Figure 16.17
x
y
1 1
1
y = x + 1 y = x + 1
Figure 16.18
28. It would be easier to integrate rst in the x direction from x = y 1 to x = y + 1, because integrating rst in the y
direction would involve two separate integrals.
_
R
(2x + 3y)
2
dA =
_
1
0
_
y+1
y1
(2x + 3y)
2
dxdy
=
_
1
0
_
y+1
y1
(4x
2
+ 12xy + 9y
2
) dxdy
=
_
1
0
_
4
3
x
3
+ 6x
2
y + 9xy
2
_
y+1
y1
dy
=
_
1
0
[
8
3
(y + 1)
3
+ 9y
2
(2y + 2)] dy
=
_

2
3
(y + 1)
4

9
2
y
4
+ 6y
3
_
1
0
=
2
3
(1)
9
2
+ 6 =
13
6
See Figure 16.18.
16.2 SOLUTIONS 1265
Problems
29. The region is bounded by x = 1, x = 4, y = 2, and y = 2x. Thus
Volume =
_
4
1
_
2x
2
(6x
2
y) dydx.
To evaluate this integral, we evaluate the inside integral rst:
_
2x
2
(6x
2
y) dy = (3x
2
y
2
)

2x
2
= 3x
2
(2x)
2
3x
2
(2
2
) = 12x
4
12x
2
.
Therefore, we have
_
4
1
_
2x
2
(6x
2
y) dydx =
_
4
1
(12x
4
12x
2
) dx =
_
12
5
x
5
4x
3
_

4
1
= 2203.2.
The volume of this object is 2203.2.
30. To nd the average value, we evaluate the integral
_
3
0
_
6
0
(x
2
+ 4y) dydx,
and then divide by the area of the base region.
To evaluate this integral, we evaluate the inside integral rst:
_
6
0
(x
2
+ 4y) dy = (x
2
y + 2y
2
)

y=6
y=0
= 6x
2
+ 72.
Therefore, we have
_
3
0
_
6
0
(x
2
+ 4y)dydx =
_
3
0
(6x
2
+ 72)dx = (2x
3
+ 72x)

3
0
= 270.
The value of the integral is 270. The area of the base region is 3 6 = 18. To nd the average value of the function, we
divide the value of the integral by the area of the base region:
Average value =
1
Area
_
3
0
_
6
0
(x
2
+ 4y) dydx =
1
18
270 = 15.
The average value is 15. This is reasonable, since the smallest value of f(x, y) on this region is 0, and the largest value is
3
2
+ 4 6 = 33.
31. To nd the average value, we rst nd the value of the integral
_
4
0
_
3
0
(xy
2
) dydx.
We evaluate the inside integral rst:
_
3
0
(xy
2
) dy =
_
xy
3
3
_

y=3
y=0
= 9x.
Therefore, we have
_
4
0
_
3
0
(xy
2
) dydx =
_
4
0
(9x) dx =
_
9x
2
2
_

4
0
= 72.
The value of the integral is 72. To nd the average value, we divide the value of the integral by the area of the region:
Average value =
1
Area
_
4
0
_
3
0
(xy
2
) dydx =
72
3 4
= 6.
The average value of f(x, y) on this rectangle is 6. This is reasonable since the smallest value of xy
2
on this region is 0
and the largest value is 4 3
2
= 36.
1266 Chapter Sixteen /SOLUTIONS
32. (a) See Figure 16.19.
y = x x + y = 1
(1/2, 1/2)
R
x
y
Figure 16.19
(b) If we integrate with respect to x rst, we have
_
R
f(x, y) dA =
_
1/2
0
_
1y
y
f(x, y) dxdy.
If we integrate with respect to y rst, the integral must be split into two parts, so
_
R
f(x, y) dA =
_
1/2
0
_
x
0
f(x, y) dy dx +
_
1
1/2
_
1x
0
f(x, y) dy dx.
(c) If f(x, y) = x,
_
R
xdA =
_
1/2
0
_
1y
y
xdxdy =
_
1/2
0
x
2
2

1y
y
dy
=
1
2
_
1/2
0
(1 y)
2
y
2
dy =
1
2
_
1/2
0
1 2y dy
=
1
2
(y y
2
)

1/2
0
=
1
8
.
Alternatively,
_
R
xdA =
_
1/2
0
_
x
0
xdy dx +
_
1
1/2
_
1x
0
xdy dx
=
_
1/2
0
xy

x
0
dx +
_
1
1/2
xy

1x
0
dx
=
_
1/2
0
x
2
dx +
_
1
1/2
x(1 x) dx
=
x
3
3

1/2
0
+
_
x
2
2

x
3
3
_

1
1/2
=
1
24
+
1
2

1
3

1
8
+
1
24
=
1
8
.
33. (a) The line x = y/2 is the line y = 2x, and y = x and y = 2x intersect at x = 0. Thus, R is the shaded region in
Figure 16.20. One expression for the integral is
_
R
f dA =
_
3
0
_
2x
x
x
2
e
x
2
dy dx.
Another expression is obtained by reversing the order of integration. When we do this, it is necessary to split R into
two regions on a line parallel to the x-axis along the point of intersection of y = x and x = 3; this line is y = 3.
Then we obtain
_
R
f dA =
_
3
0
_
y
y/2
x
2
e
x
2
dxdy +
_
6
3
_
3
y/2
x
2
e
x
2
dxdy.
16.2 SOLUTIONS 1267
(b) We evaluate the rst integral. Integrating with respect to y rst:
_
3
0
_
2x
x
x
2
e
x
2
dy dx =
_
3
0
(x
2
e
x
2
y)

2x
x
dx =
_
3
0
x
3
e
x
2
dx.
We use integration by parts with u = x
2
, v

= xe
x
2
. Then u

= 2x and v =
1
2
e
x
2
, so
_
3
0
x
3
e
x
2
dx =
1
2
x
2
e
x
2

3
0

_
3
0
xe
x
2
dx =
1
2
x
2
e
x
2

3
0

1
2
e
x
2

3
0
=
_
1
2
(9)e
9
0
_

_
1
2
e
9

1
2
_
=
1
2
+ 4e
9
.
1 2 3
1
2
3
4
5
6
7
(3, 3)
(3, 6)
y = 2x
y = x
x
y
Figure 16.20
34. As given, the region of integration is as shown in Figure 16.21. Reversing the limits gives
_
1
0
_
x
0
e
x
2
dydx =
_
1
0
_
ye
x
2

x
0
_
dx =
_
1
0
xe
x
2
dx
=
e
x
2
2

1
0
=
e 1
2
.
1
1
x
y
x = y
x = 1
Figure 16.21
x
y
y = x
1
Figure 16.22
1268 Chapter Sixteen /SOLUTIONS
35. The function sin (x
2
) has no elementary antiderivative, so we try integrating with respect to y rst. The region of integra-
tion is shown in Figure 16.22. Changing the order of integration, we get
_
1
0
_
1
y
sin (x
2
) dx dy =
_
1
0
_
x
0
sin (x
2
) dy dx
=
_
1
0
sin (x
2
) y

x
0
dx
=
_
1
0
sin (x
2
) x dx
=
cos (x
2
)
2

1
0
=
cos 1
2
+
1
2
=
1
2
(1 cos 1) = 0.23.
36. As given, the region of integration is as shown in Figure 16.23.
Reversing the limits gives
_
1
0
_
x
2
0
_
2 +x
3
dydx =
_
1
0
(y
_
2 +x
3

x
2
0
) dx
=
_
1
0
x
2
_
2 +x
3
dx
=
2
9
(2 +x
3
)
3
2

1
0
=
2
9
(3

3 2

2).
x
y
1
1
x = 1
x =

y
Figure 16.23
9
x
y
3
x = 9
x = y
2
Figure 16.24
37. As given, the region of integration is as shown in Figure 16.24.
Reversing the limits gives
_
9
0
_

x
0
y sin (x
2
) dydx =
_
9
0
_
y
2
sin (x
2
)
2

x
0
_
dx
=
1
2
_
9
0
xsin (x
2
) dx
=
cos (x
2
)
4

9
0
=
1
4

cos (81)
4
= 0.056.
16.2 SOLUTIONS 1269
38. The region of the integration is shown in Figure 16.25. To make the integration easier, we want to change the order of the
integration and get
_
1
0
_
e
e
y
x
ln x
dx dy =
_
e
1
_
ln x
0
x
ln x
dy dx
=
_
e
1
x
ln x
y

ln x
0
dx
=
_
e
1
x dx =
x
2
2

e
1
=
1
2
(e
2
1).
x
y
1
e
(e, 1)
Figure 16.25
4 4
8
x
y

y = 2x + 8

y = 2x + 8
Figure 16.26
39. Order reversed:
_
8
0
_
(8y)/2
(y8)/2
f(x, y) dxdy. See Figure 16.26.
40. (a) We divide the base region into four subrectangles as shown in Figure 16.27. The height of the object at each point
(x, y) is given by f(x, y) = xy, we label each corner of the subrectangles with the value of the function at that point.
(See Figure 16.27.) Since Volume = Height Length Width, and x = 2 and y = 3, we have
Overestimate = (12 + 24 + 6 + 12)(2)(3) = 324,
and
Underestimate = (0 + 6 + 0 + 0)(2)(3) = 36.
We average these to obtain
Volume
324 + 36
2
= 180.
2 4
3
6
f = 0
f = 0
f = 0
f = 0
f = 6
f = 12
f = 0
f = 12
f = 24
x
y
Figure 16.27
(b) We have f(x, y) = xy, so
Volume =
_
4
0
_
6
0
xy dydx =
_
4
0
_
xy
2
2
_

y=6
y=0
dx =
_
4
0
18xdx = 9x
2

4
0
= 144.
The volume of this object is 144. Notice that 144 is between the over- and underestimates, 324 and 36, found in
part (a).
1270 Chapter Sixteen /SOLUTIONS
41. (a) The contour f(x, y) = 1 lies in the xy-plane and has equation
2e
(x1)
2
y
2
= 1,
so
(x 1)
2
y
2
= ln(1/2)
(x 1)
2
+y
2
= ln 2 = 0.69.
This is the equation of a circle centered at (1, 0) in the xy-plane.
Other contours are of the form
2e
(x1)
2
y
2
= c
(x 1)
2
y
2
= ln(c/2).
Thus, all the contours are circles centered at the point (1, 0).
(b) The cross-section has equation z = f(1, y) = e
y
2
. If x = 1, the base region in the xy-plane extends from
y =

3 to y =

3. See Figure 16.28, which shows the circular region below W in the xy-plane. So
Area =
_

3

3
e
y
2
dy.
(c) Slicing parallel to the y-axis, we get
Volume =
_
2
2
_

4x
2

4x
2
e
(x1)
2
y
2
dy dx.
2 2
2
2
(1,

3) y =

4 x
2
y =

4 x
2
(1,

3)
x
y
Figure 16.28: Region beneath W in the
xy-plane
42. The intersection of the graph of f(x, y) = 25 x
2
y
2
and xy-plane is a circle x
2
+y
2
= 25. The given solid is shown
in Figure 16.29.
Thus the volume of the solid is
V =
_
R
f(x, y) dA
=
_
5
5
_

25y
2

25y
2
(25 x
2
y
2
) dx dy.
16.2 SOLUTIONS 1271
x y
z
f(x, y) = 25 x
2
y
2
Figure 16.29
x
y
z

f(x, y) = 25 x
2
y
2
z = 16
Figure 16.30
43. The intersection of the graph of f(x, y) = 25 x
2
y
2
and the plane z = 16 is a circle, x
2
+y
2
= 3
2
. The given solid
is shown in Figure 16.30.
Thus, the volume of the solid is
V =
_
R
(f(x, y) 16) dA
=
_
3
3
_

9y
2

9y
2
(9 x
2
y
2
) dx dy.
44. The solid is shown in Figure 16.31, and the base of the integral is the triangle as shown in Figure 16.32.
x
y
z
2
4
4
'
y x = 4
backside

2x + y + z = 4
E
y = 0
Figure 16.31
4 2
4
x
y
y x = 4 2x + y = 4
Figure 16.32
Thus, the volume of the solid is
V =
_
R
z dA
=
_
R
(4 2x y) dA
=
_
4
0
_
(4y)/2
y4
(4 2x y) dx dy.
1272 Chapter Sixteen /SOLUTIONS
45.
Volume =
_
2
0
_
2
0
xy dy dx =
_
2
0
1
2
xy
2

2
0
dx
=
_
2
0
2xdx
= x
2

2
0
= 4
46. The region of integration is shown in Figure 16.33. Thus
Volume =
_
1
0
_
x
0
(x
2
+y
2
) dy dx =
_
1
0
_
x
2
y +
y
3
3
_

y=x
y=0
dx =
_
1
0
4
3
x
3
dx =
x
4
3

1
0
=
1
3
.
1
1
x
y
x = y
x = 1
Figure 16.33
9
x
y
3
x = 9
x = y
2
Figure 16.34
47. The region of integration is shown in Figure 16.34. Thus,
Volume =
_
9
0
_

x
0
(x +y) dy dx =
_
9
0
_
xy +
y
2
2
_

y=

x
y=0
dx
=
_
9
0
_
x
3/2
+
x
2
_
dx =
_
2
5
x
5/2
+
x
2
4
_

9
0
=
2349
20
= 117.45.
48. The plane 2x + y + z = 4 cuts the xy-plane in the line 2x + y = 4, so the region of integration is the triangle shown in
Figure 16.35. We want to nd the volume under the graph of z = 4 2x y. Thus,
Volume =
_
2
0
_
2x+4
0
(4 2x y) dy dx =
_
2
0
_
4y 2xy
y
2
2
_

2x+4
0
dx
=
_
2
0
_
4(2x + 4) 2x(2x + 4)
(2x + 4)
2
2
_
dx
=
_
2
0
(2x
2
8x + 8) dx =
_
2
3
x
3
4x
2
+ 8x
_

2
0
=
16
3
.
16.2 SOLUTIONS 1273
2
4
x
y
x = (y 4)/2 or y = 2x + 4
Figure 16.35
49. Let R be the triangle with vertices (1, 0), (2, 2) and (0, 1). Note that (3x + 2y + 1) (x + y) = 2x + y + 1 > 0 for
x, y > 0, so z = 3x + 2y + 1 is above z = x +y on R. We want to nd
Volume =
_
R
((3x + 2y + 1) (x +y)) dA =
_
R
(2x +y + 1) dA.
We need to express this in terms of double integrals.
1 2
1
2
x
y
R
1
R
2
y = 1 + 0.5x
y = 1 x
y = 2x 2
(2, 2)
O
Figure 16.36
To do this, divide R into two regions with the line x = 1 to make regions R1 for x 1 and R2 for x 1. See
Figure 16.36. We want to nd
_
R
(2x +y + 1) dA =
_
R
1
(2x +y + 1) dA+
_
R
2
(2x +y + 1) dA.
Note that the line connecting (0, 1) and (1, 0) is y = 1 x, and the line connecting (0, 1) and (2, 2) is y = 1 +0.5x. So
_
R
1
(2x +y + 1) dA =
_
1
0
_
1+0.5x
1x
(2x +y + 1) dy dx.
The line between (1, 0) and (2, 2) is y = 2x 2, so
_
R
2
(2x +y + 1) dA =
_
2
1
_
1+0.5x
2x2
(2x +y + 1) dy dx.
1274 Chapter Sixteen /SOLUTIONS
We can now compute the double integral for R1:
_
1
0
_
1+0.5x
1x
(2x +y + 1) dy dx =
_
1
0
_
2xy +
y
2
2
+y
_

1+0.5x
1x
dx
=
_
1
0
_
21
8
x
2
+ 3x
_
dx
=
_
7
8
x
3
+
3
2
x
2
_

1
0
dx
=
19
8
,
and the double integral for R2:
_
2
1
_
1+0.5x
2x2
(2x +y + 1) dy dx =
_
2
1
(2xy +y
2
/2 +y)

1+0.5x
2x2
dx
=
_
1
0
_

39
8
x
2
+ 9x +
3
2
_
dx
=
_

13
8
x
3
+
9
2
x
2
+
3
2
x
_

2
1
=
29
8
.
So, Volume =
19
8
+
29
8
=
48
8
= 6.
50. The region R is shaded in Figure 16.37. The integral is
_
R
xy dA =
_
a
0
_

a
2
y
2
ay
xy dxdy
=
_
a
0
1
2
x
2
y

a
2
y
2
ay
dy =
1
2
_
a
0
((a
2
y
2
)y (a y)
2
y)y dy
=
1
2
_
a
0
(2ay
2
2y
3
) dy =
_
ay
3
3

y
4
4
_

a
0
=
a
4
12
.
a
a
'
x + y = a

x
2
+ y
2
= a
2
x
y
Figure 16.37
x
y
z
1/a
1/c
1/b
ax +
by =
1
Figure 16.38
51. We want to calculate the volume of the tetrahedron shown in Figure 16.38.
We rst nd the region in the xy-plane where the graph of ax + by + cz = 1 is above the xy-plane. When z = 0
we have ax + by = 1. So the region over which we want to integrate is bounded by x = 0, y = 0 and ax + by = 1.
Integrating with respect to y rst, we have
Volume =
_
1/a
0
_
(1ax)/b
0
z dy dx =
_
1/a
0
_
(1ax)/b
0
1 by ax
c
dy dx
16.2 SOLUTIONS 1275
=
_
1/a
0
_
y
c

by
2
2c

axy
c
_

y=(1ax)/b
y=0
dx
=
_
1/a
0
1
2bc
(1 2ax +a
2
x
2
) dx
=
1
6abc
.
52. The region bounded by the x-axis and the graph of y = x x
2
is shown in Figure 16.39. The area of this region is
A =
_
1
0
(x x
2
)dx = (
x
2
2

x
3
3
)

1
0
=
1
2

1
3
=
1
6
.
1
2
1
0 x
y
1
4
Figure 16.39
So the average distance to the x-axis for points in the region is
Average distance =
_
R
y dA
area(R)
_
R
y dA =
_
1
0
_
_
xx
2
0
y dy
_
dx
=
_
1
0
_
x
2
2
x
3
+
x
4
2
_
dx =
1
6

1
4
+
1
10
=
1
60
.
Therefore the average distance is
1/60
1/6
= 1/10.
53. Assume the length of the two legs of the right triangle are a and b, respectively. See Figure 16.40. The line through (a, 0)
and (0, b) is given by
y
b
+
x
a
= 1. So the area of this triangle is
A =
1
2
ab.
x
y
b
a
Figure 16.40
1276 Chapter Sixteen /SOLUTIONS
Thus the average distance from the points in the triangle to the y-axis (one of the legs) is
Average distance =
1
A
_
a
0
_

b
a
x+b
0
xdy dx
=
2
ab
_
a
0
_

b
a
x
2
+bx
_
dx
=
2
ab
_

b
3a
x
3
+
b
2
x
2
_

a
0
=
2
ab
_
a
2
b
6
_
=
a
3
.
Similarly, the average distance from the points in the triangle to the x-axis (the other leg) is
Average distance =
1
A
_
b
0
_

a
b
y+a
0
y dxdy
=
2
ab
_
b
0
_

a
b
y
2
+ay
_
dy
=
2
ab
_
ab
2
6
_
=
b
3
.
54. (a) We have
Average value of f =
1
Area of Square
_
Square
f dA
=
1
4
_
2
0
_
2
0
(ax
2
+bxy +cy
2
) dydx =
1
4
_
2
0
_
ax
2
y +bx
y
2
2
+c
y
3
3
_

y=2
y=0
dx
=
1
4
_
2
0
_
2ax
2
+ 2bx +
8
3
c
_
dx =
1
4
_
2
3
ax
3
+bx
2
+
8
3
cx
_

2
0
=
1
4
_
16
3
a + 4b +
16
3
c
_
=
4
3
a +b +
4
3
c
The average value will be 20 if and only if (4/3)a +b + (4/3)c = 20.
(b) Since (4/3)a +b + (4/3)c = 20, we must have b = 20 (4/3)a (4/3)c. Any function f(x, y) = ax
2
+ (20
(4/3)a(4/3)c)xy +cy
2
where a and c are any real numbers is a correct solution. For example, a = 1, c = 3 leads
to the function f(x, y) = x
2
+(44/3)xy +3y
2
, and a = 3, c = 0 leads to the function f(x, y) = 3x
2
+24xy,
both of which have average value 20 on the given square. See Figures 16.41 and 16.42.
x
y
Figure 16.41: f(x, y) = x
2
+
44
3
xy+3y
2
x
y
Figure 16.42: f(x, y) = 3x
2
+ 24xy
16.2 SOLUTIONS 1277
55. (a) We have
Average value of f =
1
Area of Rectangle
_
Rectangle
f dA
=
1
6
_
2
x=0
_
3
y=0
(ax +by) dydx =
1
6
_
2
0
_
axy +b
y
2
2
_

y=3
y=0
dx
=
1
6
_
2
0
_
3ax +
9
2
b
_
dx =
1
6
_
3
2
ax
2
+
9
2
bx
_

2
0
=
1
6
(6a + 9b)
= a +
3
2
b.
The average value will be 20 if and only if a + (3/2)b = 20.
This equation can also be expressed as 2a + 3b = 40, which shows that f(x, y) = ax + by has average value
of 20 on the rectangle 0 x 2, 0 y 3 if and only if f(2, 3) = 40.
(b) Since 2a +3b = 40, we must have b = (40/3) (2/3)a. Any function f(x, y) = ax +((40/3) (2/3)a)y where
a is any real number is a correct solution. For example, a = 1 leads to the function f(x, y) = x + (38/3)y, and
a = 3 leads to the function f(x, y) = 3x+(46/3)y, both of which have average value 20 on the given rectangle.
See Figure 16.43 and 16.44.
x
y
Figure 16.43: f(x, y) = x +
38
3
y
x
y
Figure 16.44: f(x, y) = 3x +
46
3
y
56. (a) One solution would be to arrange that the minimum values of f on the square occur at the corners, so that the corner
values give an underestimate of the average. See Figure 16.45.
1
1
100
98
96
94
x
y
Figure 16.45
1
1
0
0.1
0.2
0.3
x
y
Figure 16.46
(b) One solution would be to arrange that the maximum values of f on the square occur at the corners, so that the corner
values give an overestimate of the average. See Figure 16.46.
57. The force, F, acting on A, a small piece of area, is given by
F pA,
1278 Chapter Sixteen /SOLUTIONS
where p is the pressure at that point. Thus, if R is the rectangle, the total force is given by
F =
_
R
p dA.
We choose coordinates with the origin at one corner of the plate. See Figure 16.47.
a
b
(a, b)
x
y
Figure 16.47
Suppose p is proportional to the square of the distance from the corner represented by the origin. Then we have
p = k(x
2
+y
2
), for some positive constant k.
Thus, we want to compute
_
R
k(x
2
+y
2
)dA. Rewriting as an iterated integral, we have
F =
_
R
k(x
2
+y
2
) dA =
_
b
0
_
a
0
k(x
2
+y
2
) dxdy = k
_
b
0
_
x
3
3
+xy
2

a
0
_
dy
= k
_
b
0
_
a
3
3
+ay
2
_
dy = k
_
a
3
y
3
+a
y
3
3

b
0
_
=
k
3
(a
3
b +ab
3
).
Solutions for Section 16.3
Exercises
1.
_
W
f dV =
_
2
0
_
1
1
_
3
2
(x
2
+ 5y
2
z) dz dy dx
=
_
2
0
_
1
1
(x
2
z + 5y
2
z
1
2
z
2
)

3
2
dy dx
=
_
2
0
_
1
1
(x
2
+ 5y
2

5
2
) dy dx
=
_
2
0
(x
2
y +
5
3
y
3

5
2
y)

1
1
dx
=
_
2
0
(2x
2
+
10
3
5) dx
= (
2
3
x
3

5
3
x)

2
0
=
16
3

10
3
= 2
16.3 SOLUTIONS 1279
2.
_
W
f dV =
_
1
0
_
1
0
_
2
0
(ax +by +cz) dz dy dx
=
_
1
0
_
1
0
(2ax + 2by + 2c) dy dx
=
_
1
0
(2ax +b + 2c) dx
= a +b + 2c
3.
_
W
f dV =
_
a
0
_
b
0
_
c
0
e
xyz
dz dy dx
=
_
a
0
_
b
0
_
c
0
e
x
e
y
e
z
dz dy dx
=
_
a
0
_
b
0
e
x
e
y
(e
z
)

c
0
dy dx
=
_
a
0
_
b
0
e
x
e
y
(e
c
+ 1) dy dx
= (1 e
c
)
_
a
0
e
x
(e
y
)

b
0
dx
= (1 e
b
)(1 e
c
)
_
a
0
e
x
dx
= (1 e
a
)(1 e
b
)(1 e
c
)
4.
_
W
f dV =
_

0
_

0
_

0
sin xcos(y +z) dz dy dx
=
_

0
_

0
sin xsin(y +z)

0
dy dx
=
_

0
_

0
sin x[sin(y +) sin y] dy dx
=
_

0
_

0
sin x(2 sin y) dy dx
= 2
_

0
sin x(cos y)

0
dx
= 2
_

0
2 sin xdx
= 4(cos x)

0
= (4)(2) = 8
5. The region is the half cylinder in Figure 16.48.
6. The region is the half cylinder in Figure 16.49.
1280 Chapter Sixteen /SOLUTIONS
7. The region is the quarter sphere in Figure 16.50.
1
1
1
x
y
z
Figure 16.48
x
y
z
1
1
1
Figure 16.49
1
1
1
x
y
z
Figure 16.50
8. The region is the half cylinder in Figure 16.51.
9. The region is the cylinder in Figure 16.52.
10. The region is the hemisphere in Figure 16.53.
1
1
1
x
y
z
Figure 16.51
x
y
z
1
1
1
Figure 16.52
1
1
1
x
y
z
Figure 16.53
11. The region is the hemisphere in Figure 16.54.
12. The region is the quarter sphere in Figure 16.55.
13. The region is the quarter sphere in Figure 16.56.
1
1
1
x
y
z
Figure 16.54
1
1
1
x
y
z
Figure 16.55
1
1
1
x
y
z
Figure 16.56
Problems
14. Since z = x +y is below z = 1 + 2x + 2y for x, y 0, we have
V =
_
1
0
_
2
0
_
1+2x+2y
x+y
1 dz dy dx.
The order of integration of x and y can be reversed.
16.3 SOLUTIONS 1281
15. For x
2
+y
2
1, the paraboloid z = x
2
+y
2
is below the sphere x
2
+y
2
+z
2
= 4, so
V =
_
1
1
_

1x
2

1x
2
_

4x
2
y
2
x
2
+y
2
1 dz dy dx.
The order of integration of x and y can be reversed.
16. The two surfaces are planes given by
z = 6 2x 2y
z = 6 3x 4y.
For x, y 0, the plane z = 6 2x 2y is above z = 6 3x 4y. The region in the xy-plane is shown in Figure 16.57.
Thus
V =
_
1
0
_
1x
0
_
62x2y
63x4y
1 dz dy dx.
The order of integration of x and y can be reversed.
1
1
y = 1 x
x
y
Figure 16.57

5
y =

5 x
2
y =

5 x
2
x
y
Figure 16.58
17. The sphere x
2
+y
2
+z
2
= 9 intersects the plane z = 2 in the circle
x
2
+y
2
+ 2
2
= 9
x
2
+y
2
= 5.
The upper half of the sphere is given by z =
_
9 x
2
y
2
. Thus, using the limits from Figure 16.58 gives
V =
_

5

5
_

5x
2

5x
2
_

9x
2
y
2
2
1 dz dy dx.
The order of integration of x and y can be reversed.
18. The top half of the sphere is given by
z =
_
4 x
2
y
2
.
The region in the xy-plane is shown in Figure 16.59. If we integrate with respect to y rst, we have to break the region in
two pieces. Thus it is easier to integrate with respect to x rst, giving
V =
_
2
0
_
(y+2)/2
y
_

4x
2
y
2
0
1 dz dxdy.
1282 Chapter Sixteen /SOLUTIONS
1 2
2
y = x
y = 2x 2
x
y
Figure 16.59
1 2
2
x
2
+ y
2
= 4
x
y
Figure 16.60
19. The top half of the sphere is given by
z =
_
4 x
2
y
2
.
The region in the xy-plane is shown in Figure 16.60. If we integrate with respect to x rst, we have to break the region
into two pieces. Thus, it is easier to integrate with respect to y rst, giving
V =
_
1
0
_

4x
2
0
_

4x
2
y
2
0
1 dz dy dx.
20. A slice through W for a xed value of x is a semi-circle the boundary of which is y
2
= r
2
x
2
z
2
, for y 0, so the
inner integral is
_

r
2
x
2
z
2
0
f(x, y, z) dy.
Lining up these stacks parallel to z-axis gives a slice from z =

r
2
x
2
to z =

r
2
x
2
giving
_

r
2
x
2

r
2
x
2
_

r
2
x
2
z
2
0
f(x, y, z) dy dz.
Finally, there is a slice for each x between r and r, so the integral we want is
_
r
r
_

r
2
x
2

r
2
x
2
_

r
2
x
2
z
2
0
f(x, y, z) dy dz dx.
21. A slice through W for a xed value of x is a semi-circle the boundary of which is z
2
= r
2
x
2
y
2
, for z 0, so the
inner integral is
_

r
2
x
2
y
2
0
f(x, y, z) dz.
Lining up these stacks parallel to y-axis gives a slice from y =

r
2
x
2
to y =

r
2
x
2
giving
_

r
2
x
2

r
2
x
2
_

r
2
x
2
y
2
0
f(x, y, z) dz dy.
Finally, there is a slice for each x between 0 and r, so the integral we want is
_
r
0
_

r
2
x
2

r
2
x
2
_

r
2
x
2
y
2
0
f(x, y, z) dz dy dx.
16.3 SOLUTIONS 1283
22. A slice through W for a xed value of y is a semi-circle the boundary of which is z
2
= r
2
x
2
, for z 0, so the inner
integral is
_

r
2
x
2
0
f(x, y, z) dz.
Lining up these stacks parallel to x-axis gives a slice from x = r to x = r giving
_
r
r
_

r
2
x
2
0
f(x, y, z) dz dx.
Finally, there is a slice for each y between 0 and 1, so the integral we want is
_
1
0
_
r
r
_

r
2
x
2
0
f(x, y, z) dz dxdy.
23. A slice through W for a xed value of x is a semi-circle the boundary of which is y
2
= 4 z
2
, for y 0, so the inner
integral is
_

4z
2
0
f(x, y, z) dy.
Lining up these stacks parallel to z-axis gives a slice from z = 2 to z = 2 giving
_
2
2
_

4z
2
0
f(x, y, z) dy dz.
Finally, there is a slice for each x between 0 and 1, so the integral we want is
_
1
0
_
2
2
_

4z
2
0
f(x, y, z) dy dz dx.
24. The required volume, V , is given by
V =
_
10
0
_
10x
0
_
10
x+y
dzdydx
=
_
10
0
_
10x
0
(10 (x +y)) dydx
=
_
10
0
_
10y xy
1
2
y
2
_
y=10x
y=0
dx
=
_
10
0
1
2
(10 x)
2
dx
=
500
3
25. The pyramid is shown in Figure 16.61. The planes y = 0, and y x = 4, and 2x+y +z = 4 intersect the plane z = 6
in the lines y = 0, y x = 4, 2x +y = 10 on the z = 6 plane as shown in Figure 16.62.
These three lines intersect at the points (4, 0, 6), (5, 0, 6), and (2, 6, 6). Let R be the triangle in the planes
z = 6 with the above three points as vertices. Then, the volume of the solid is
V =
_
6
0
_
(10y)/2
y4
_
42xy
6
dz dxdy
=
_
6
0
_
(10y)/2
y4
(10 2x y) dxdy = 162
=
_
6
0
(10x x
2
xy)

(10y)/2
y4
dy
1284 Chapter Sixteen /SOLUTIONS
=
_
6
0
(
9y
2
4
27y + 81) dy
= 162
x
y
z
2
4
4
'
2x + y + z = 4
E
y = 0
'
z = 6 bottom
'
y x = 4 backside
(4, 0, 6)
(5, 0, 6)
(2, 6, 6)
Figure 16.61
(4, 0, 6) (5, 0, 6)
(2, 6, 6)
z = 6 plane
x
y
y x = 4 2x + y 6 = 4
Figure 16.62
26. Figure 16.63 shows a slice through the region for a xed x. The required volume, V , is given by
V =
_
2
1
_
1
0
_
3y
y
dz dy dx =
_
2
1
_
1
0
z

3y
y
dy dx =
_
2
1
_
1
0
2y dy dx
=
_
2
1
y
2

1
0
dx =
_
2
1
dx = 1.
1
0
1
2
3
z = y
z = 3y
y
z
Figure 16.63
1
0
1
z = x
z = x
2
x
z
Figure 16.64
27. Figure 16.64 shows a slice through the region for a xed value of y. We break the region into small cubes of volume
V = xyz. A stack of cubes vertically above the point (x, z) in the xz-plane gives the strip shown in Figure 16.64
and so the inner integral is
_
x
x
2
dz
16.3 SOLUTIONS 1285
The plane and the surface meet when x = x
2
, giving x(1 x) = 0, so x = 0 or x = 1. Lining up the stacks parallel to
the z-axis gives a slice from x = 0 to x = 1. Thus, the limits on the middle integral are
_
1
0
_
x
x
2
dz dx.
Finally, there is a slice for each y between 0 and 3, so the integral we want is
_
3
0
_
1
0
_
x
x
2
dz dxdy.
The required volume, V , is given by
V =
_
3
0
_
1
0
_
x
x
2
dz dxdy
=
_
3
0
_
1
0
_
x x
2
_
dxdy
=
_
3
0
x
2
2

x
3
3

1
0
dy
=
_
3
0
1
6
dy
=
1
6
_
3
0
dy
=
1
6
3
=
1
2
.
28. The required volume, V , is given by
V =
_
5
0
_
5x
0
_
x+y
0
dz dy dx
=
_
5
0
_
5x
0
(x +y) dy dx
=
_
5
0
xy +
1
2
y
2

y=5x
y=0
dx
=
_
5
0
_
x(5 x) +
1
2
(5 x)
2
_
dx
=
125
3
.
29. The required volume, V , is given by
V =
_
5
0
_
3
0
_
x
2
0
dz dy dx
=
_
5
0
_
3
0
x
2
dy dx
=
_
5
0
x
2
y

y=3
y=0
dx
=
_
5
0
3x
2
dx
= 125.
1286 Chapter Sixteen /SOLUTIONS
30. Since x +y +z = 1 can be written as
z = 1 x y,
the plane z = 1 +x +y is above the plane z = 1 x y for x 0, y 0. The region of integration in the xy-plane is
the triangle shown in Figure 16.65. Thus
Volume =
_
1
0
_
1x
0
_
1+x+y
1xy
1 dz dy dx =
_
1
0
_
1x
1
z

1+x+y
1xy
dy dx
=
_
1
0
_
1x
0
((1 +x +y) (1 x y)) dy dx =
_
1
0
_
1x
0
(2x + 2y) dy dx =
_
1
0
2xy +y
2

1x
0
dx
=
_
1
0
(2x(1 x) + (1 x)
2
) dx =
_
1
0
(1 x
2
) dx =
_
x
x
3
3
_

1
0
=
2
3
.
1
1
y = 1 x
x
y
Figure 16.65
31. The plane x + y + z = 1 cuts the xy-plane in the line x + y = 1. For x + y < 1, the plane x + y + z = 1 is above the
xy-plane. For 1 < x + y 2, the plane x + y + z = 1 is below the xy-plane. Therefore, z = 1 x y is positive for
x +y < 1 and negative for 1 < x +y < 2. Thus
Volume =
_
x+y1
(1 x y) dA
_
1x+y2
(1 x y) dA
From Figure 16.66, we see that the region 1 x + y 2 must be split into two, for example as shown. Integrating with
respect to x rst, we have
Volume =
_
1
0
_
1y
0
(1 x y) dxdy
__
1
0
_
2y
1y
(1 x y) dxdy +
_
2
1
_
2y
0
(1 x y) dxdy
_
=
_
1
0
_
x
x
2
2
xy
_

1y
0
dy
_
1
0
_
x
x
2
2
xy
_

2y
1y
dy
_
2
1
_
x
x
2
2
xy
_

2y
0
dy
=
_
1
0
1 y
2
(2 (1 y) 2y) dy
_
1
0
_
2 y
2
(2 (2 y) 2y)
1 y
2
(2 (1 y) 2y)
_
dy

_
2
1
2 y
2
(2 (2 y) 2y) dy
=
_
1
0
(1 y)
2
2
dy +
_
1
0
1
2
dy
_
2
1
y(y 2)
2
dy
=
(1 y)
3
6

1
0
+
y
2

1
0

1
2
_
y
3
3
y
2
_

2
1
=
1
6
+
1
2
+
1
3
= 1.
16.3 SOLUTIONS 1287
1 2
1
2
x = 1 y
x = 2 y
x
y
Figure 16.66
32. The region looks like an upside-down trough, with cross section in the xz-plane shown in Figure 16.67, extending a
distance of 3 cm in the y-direction.
Since the region and the density function are symmetric about the z-axis, we nd the mass of the right side and
double it. In grams
Mass = 2
_
3
0
_
1
0
_
1x
0
(10 z) dz dxdy
= 2
_
3
0
_
1
0
_
10z
z
2
2
_

1x
0
dxdy
= 2
_
3
0
_
1
0
_
10(1 x)
(1 x)
2
2
_
dxdy
=
_
3
0
_
1
0
(19 18x x
2
) dxdy
= 3
_
19x 9x
2

x
3
3
_

1
0
= 3
_
19 9
1
3
_
= 29 gm.
1 1
1
z = 1 + x z = 1 x
x
z
Figure 16.67
33. (a) The vectors u =

j and v =

k lie in the required plane so p = u v =

i +

j +

k is perpendicular
to this plane. Let (x, y, z) be a point in the plane, then (x 1)

i + y

j + z

k is perpendicular to p , so ((x 1)

i +
y

j +z

k ) (

i +

j +

j ) = 0 and so
(x 1) +y +z = 0.
Therefore, the equation of the required plane is x +y +z = 1.
(b) The required volume, V , is given by
V =
_
1
0
_
1x
0
_
1xy
0
dz dy dx
=
_
1
0
_
1x
0
(1 x y) dy dx
1288 Chapter Sixteen /SOLUTIONS
=
_
1
0
y xy
1
2
y
2

1x
0
dx
=
_
1
0
_
1 x x(1 x)
1
2
(1 x)
2
_
dx
=
_
1
0
1
2
(1 x)
2
dx
=
1
6
.
34. (a) The equation of the surface of the whole cylinder along the y-axis is x
2
+z
2
= 1. The part we want is
z =
_
1 x
2
0 y 10.
See Figure 16.68.
x
1
y
10
z
Figure 16.68
(b) The integral is
_
D
f(x, y, z) dV =
_
10
0
_
1
1
_

1x
2
0
f(x, y, z) dzdxdy.
35. The region of integration is shown in Figure 16.69, and the mass of the given solid is given by
x
y
z
3
2
6
x
3
+
y
2
+
z
6
= 1
or z = 2x 3y + 6
'
x
3
+
y
2
= 1
or y =
2
3
x + 2
Figure 16.69
mass =
_
R
dV
=
_
3
0
_

2
3
x+2
0
_
2x3y+6
0
(x +y) dzdydx
16.3 SOLUTIONS 1289
=
_
3
0
_

2
3
x+2
0
(x +y)z

2x3y+6
0
dydx
=
_
3
0
_

2
3
x+2
0
(x +y)(2x 3y + 6) dydx
=
_
3
0
_

2
3
x+2
0
(2x
2
3y
2
5xy + 6x + 6y) dydx
=
_
3
0
_
2x
2
y y
3

5
2
xy
2
+ 6xy + 3y
2
_

2
3
x+2
0
dx
=
_
3
0
_
14
27
x
3

8
3
x
2
+ 2x + 4
_
dx
=
_
7
54
x
4

8
9
x
3
+x
2
+ 4x
_

3
0
=
7
54
3
4

8
9
3
3
+ 3
2
+ 12 =
21
2
3 =
15
2
.
36. The pyramid is shown in Figure 16.70. The planes y = 0, and y x = 4, and 2x+y +z = 4 intersect the plane z = 6
in the lines y = 0, y x = 4, 2x +y = 10 as shown in Figure 16.71.
x
y
z
2
4
4
'
2x + y + z = 4
E
y = 0
'
z = 6 bottom
'
y x = 4 backside
(4, 0, 6)
(5, 0, 6)
(2, 6, 6)
Figure 16.70
(4, 0, 6) (5, 0, 6)
(2, 6, 6)
z = 6 plane
x
y
y x = 4 2x + y 6 = 4
Figure 16.71
These three lines (the edges of the pyramid) intersect the plane z = 6 at the points (4, 0, 6), (5, 0, 6), and
(2, 6, 6). Let R be the triangle in the plane z = 6 with these three points as vertices. Then, the mass of the solid is
Mass =
_
6
0
_
(10y)/2
y4
_
42xy
6
(x, y, z) dz dxdy
=
_
6
0
_
(10y)/2
y4
_
42xy
6
y dz dxdy
=
_
6
0
_
(10y)/2
y4
y(10 2x y) dxdy
=
_
6
0
y(10x x
2
xy)

x=(10y)/2
x=y4
dy
=
_
6
0
(
9y
3
4
27y
2
+ 81y) dy
= 243.
1290 Chapter Sixteen /SOLUTIONS
37. From the problem, we know that (x, y, z) is in the cube which is bounded by the three coordinate planes, x = 0, y = 0,
z = 0 and the planes x = 2, y = 2, z = 2. We can regard the value x
2
+y
2
+z
2
as the density of the cube. The average
value of x
2
+y
2
+z
2
is given by
average value =
_
V
(x
2
+y
2
+z
2
) dV
volume(V )
=
_
2
0
_
2
0
_
2
0
(x
2
+y
2
+z
2
) dxdydz
8
=
_
2
0
_
2
0
_
x
3
3
+ (y
2
+z
2
)x
_

2
0
dydz
8
=
_
2
0
_
2
0
_
8
3
+ 2y
2
+ 2z
2
_
dydz
8
=
_
2
0
_
8
3
y +
2
3
y
3
+ 2z
2
y
_

2
0
dz
8
=
_
2
0
_
16
3
+
16
3
+ 4z
2
_
dz
8
=
_
32
3
z +
4
3
z
3
_

2
0
8
=
_
64
3
+
32
3
_
8
= 4.
38. Positive. The function
_
x
2
+y
2
is positive, so its integral over the solid W is positive.
39. Positive. If (x, y, z) is any point inside the solid W then
_
x
2
+y
2
< z. Thus the integrand z
_
x
2
+y
2
> 0, and so
its integral over the solid W is positive.
40. Zero. The value of x is positive above the rst and fourth quadrants in the xy-plane, and negative (and of equal absolute
value) above the second and third quadrants. The integral of x over the entire solid cone is zero because the integrals over
the two halves of the cone cancel.
41. Zero. The value of y is positive on the half of the cone above the second and third quadrants and negative (of equal
absolute value) on the half of the cone above the third and fourth quadrants. The integral of y over the entire solid cone is
zero because the integrals over the four quadrants cancel.
42. Positive. Since z is positive on W, its integral is positive.
43. Zero. You can see this in several ways. One way is to observe that xy is positive on part of the cone above the rst and
third quadrants (where x and y are of the same sign) and negative (of equal absolute value) on the part of the cone above
the second and fourth quadrants (where x and y have opposite signs). These add up to zero in the integral of xy over all
of W.
Another way to see that the integral is zero is to write the triple integral as an iterated integral, say integrating rst
with respect to x. For xed y and z, the x-integral is over an interval symmetric about 0. The integral of x over such an
interval is zero. If any of the inner integrals in an iterated integral is zero, then the triple integral is zero.
44. Zero. Write the triple integral as an iterated integral, say integrating rst with respect to x. For xed y and z, the x-integral
is over an interval symmetric about 0. The integral of x over such an interval is zero. If any of the inner integrals in an
iterated integral is zero, then the triple integral is zero.
45. Negative. If (x, y, z) is any point inside the cone then z < 2. Hence the function z 2 is negative on W and so is its
integral.
46. Positive. The function e
xyz
is a positive function everywhere so its integral over W is positive.
47. Positive. The function
_
x
2
+y
2
is positive, so its integral over the solid W is positive.
48. Positive. If (x, y, z) is any point inside the solid W then
_
x
2
+y
2
< z. Thus z
_
x
2
+y
2
> 0, and so its integral
over the solid W is positive.
49. Positive. The value of x is positive on the half-cone, so its integral is positive.
16.3 SOLUTIONS 1291
50. Zero. y is positive on the half of the half-cone above the rst quadrant in the xy-plane and negative (of equal absolute
value) on the half of the half-cone above the fourth quadrant. The integral of y over W is zero because the integrals over
each half add up to zero.
51. Positive. Since z is positive on W, its integral is positive.
52. Zero. You can see this in several ways. One way is to observe that xy is positive on part of the cone above the rst
quadrant (where x and y are of the same sign) and negative (of equal absolute value) on the part of the cone above the
fourth quadrant (where x and y have opposite signs). These add up to zero in the integral of xy over all of W.
Another way to see that the integral is zero is to write the triple integral as an iterated integral, say integrating rst
with respect to y. For xed x and z, the y-integral is over an interval symmetric about 0. The integral of y over such an
interval is zero. If any of the inner integrals in an iterated integral is zero, then the triple integral is zero.
53. Zero. Write the triple integral as an iterated integral, say integrating rst with respect to y. For xed x and z, the y-integral
is over an interval symmetric about 0. The integral of y over such an interval is zero. If any of the inner integrals in an
iterated integral is zero, then the triple integral is zero.
54. Negative. If (x, y, z) is any point inside the cone then z < 2. Hence the function z 2 is negative on W and so is its
integral.
55. Positive. The function e
xyz
is a positive function everywhere so its integral over W is positive.
56. Orient the region as shown in Figure 16.72 and use Cartesian coordinates with origin at the center of the sphere. The
equation of the sphere is x
2
+ y
2
+ z
2
= 25, and we want the volume between the planes z = 3 and z = 5. The plane
z = 3 cuts the sphere in the circle x
2
+y
2
+ 3
2
= 25, or x
2
+y
2
= 16.
Volume =
_
4
4
_

16x
2

16x
2
_

25x
2
y
2
3
dzdydx.
x
y
z
5

Sphere is
x
2
+ y
2
+ z
2
= 25
s
x
2
+ y
2
= 16
s
Circle is
x
2
+ y
2
= 16
T
c
3
T
c
2
Figure 16.72
x
y
z
Figure 16.73
57. The intersection of two cylinders x
2
+z
2
= 1 and y
2
+z
2
= 1 is shown in Figure 16.73. This region is bounded by four
surfaces:
z =
_
1 x
2
, z =
_
1 x
2
, y =
_
1 z
2
, and y =
_
1 z
2
So the volume of the given solid is
V =
_
1
1
_

1x
2

1x
2
_

1z
2

1z
2
dy dz dx
58. Set up axes as in Figure 16.74.
1292 Chapter Sixteen /SOLUTIONS
2
2
2
x
y
z
Figure 16.74
The slanting plane has equation z = 2 x y and the line where it intersects the xy-plane has equation y = 2 x.
The mass of the bottom part is 1 times its volume, VBottom, where
VBottom =
_
2
0
_
2x
0
_
2xy
0
dz dy dx =
_
2
0
_
2x
0
z

2xy
0
dy dx
=
_
2
0
_
2x
0
(2 x y) dy dx =
_
2
0
_
(2 x)y
y
2
2
_

2x
0
dx
=
_
2
0
(2 x)
2
2
dx =
(2 x)
3
6

2
0
=
4
3
m
3
.
Since the cube has volume 2
3
= 8 m
3
, the upper part has volume VTop = 8 4/3 = 20/3 m
3
. Thus
Mass = VBottom1 +VTop2 =
4
3
1 +
20
3
2 gm.
59. The mass m is given by
m =
_
W
1 dV =
_
1
0
_
1
0
_
x+y+1
0
1 dz dy dx
=
_
1
0
_
1
0
(x +y + 1) dy dx
=
_
1
0
_
xy +y
2
/2 +y)

1
0
dx
=
_
1
0
(x + 3/2) dx = 2 gm.
Then the x-coordinate of the center of mass is given by
x =
1
2
_
W
xdV =
1
2
_
1
0
_
1
0
_
x+y+1
0
xdz dy dx
=
1
2
_
1
0
_
1
0
x(x +y + 1) dy dx
=
1
2
_
1
0
_
x
2
y +xy
2
/2 +xy)

1
0
dx
=
1
2
_
1
0
(x
2
+ 3/2x) dx = 13/24 cm.
An essentially identical calculation (since the region is symmetric in x and y) gives y = 13/24 cm.
16.3 SOLUTIONS 1293
Finally, we compute z:
z =
1
2
_
W
z dV =
1
2
_
1
0
_
1
0
_
x+y+1
0
z dz dy dx
=
1
2
_
1
0
_
1
0
(x +y + 1)
2
/2 dy dx
=
1
2
_
1
0
(x +y + 1)
3
/6

1
0
dx
=
1
12
_
1
0
((x + 2)
3
(x + 1)
3
) dx = 25/24 cm.
So ( x, y, z) = (13/24, 13/24, 25/24).
60. The mass m is given by
m =
_
W
1 dV =
_
1
0
_
(1x)/2
0
_
(1x2y)/3
0
1 dz dy dx
=
_
1
0
_
(1x)/2
0
1 x 2y
3
dy dx
=
1
3
_
1
0
(y xy y
2
)

(1x)/2
0
dx
=
1
3
__
1
0
1 x
2
x
1 x
2

_
1 x
2
_
2
_
dx
=
1
3
_
1
0
_
(1 x)
2
2

(1 x)
2
4
_
dx
=
1
3
_
(1 x)
3
12
_

1
0
= 1/36 gm.
Then the coordinates of the center of mass are given by
x = 36
_
W
xdV = 36
_
1
0
_
(1x)/2
0
_
(1x2y)/3
0
xdz dy dx = 1/4 cm.
and
y = 36
_
W
y dV = 36
_
1
0
_
(1x)/2
0
_
(1x2y)/3
0
y dz dy dx = 1/8 cm.
and
z = 36
_
W
z dV = 36
_
1
0
_
(1x)/2
0
_
(1x2y)/3
0
z dz dy dx = 1/12 cm.
61. The volume V of the solid is 1 2 3 = 6. We need to compute
m
6
_
W
x
2
+y
2
dV =
m
6
_
1
0
_
2
0
_
3
0
x
2
+y
2
dz dy dx
=
m
6
_
1
0
_
2
0
3(x
2
+y
2
) dy dx
=
m
2
_
1
0
(x
2
y +y
3
/3)

2
0
dx
=
m
2
_
1
0
(2x
2
+ 8/3) dx = 5m/3
1294 Chapter Sixteen /SOLUTIONS
62. The volume of the solid is 8abc, so we need to evaluate
m
8abc
_
W
(y
2
+z
2
) dV =
m
8abc
_
c
c
_
b
b
_
a
a
(y
2
+z
2
) dxdy dz
=
m
8abc
_
c
c
_
b
b
2a(y
2
+z
2
) dy dz
=
m
4bc
_
c
c
(y
3
/3 +yz
2
)

b
b
dz
=
m
2c
_
c
c
(b
2
/3 +z
2
) dz
= m(b
2
+c
2
)/3
63. By the denition, we have that
a +b =
m
V
_
W
(y
2
+z
2
) dV +
m
V
_
W
(x
2
+z
2
) dV
=
m
V
_
W
(x
2
+y
2
+ 2z
2
) dV
=
m
V
_
W
(x
2
+y
2
) dV +
m
V
_
W
(2z
2
) dV
= c +
m
V
_
W
(2z
2
) dV
Since z
2
is always positive, the integral
_
W
(2z
2
) dV will be positive, thus a +b > c.
Solutions for Section 16.4
Exercises
1.
_
2
0
_

2
0
f rdr d
2.
_
/2
0
_
1/2
0
f rdr d
3.
_
3/2
/2
_
2
1
f rdr d
4.
_
3/4
/4
_
2
0
f rdr d
5. A circle is best described in polar coordinates. The radius is 5, so r goes from 0 to 5. To include the entire circle, we need
to go from 0 to 2. The integral is
_
2
0
_
5
0
f(r cos , r sin ) r dr d.
6. Since this is a triangular region we can use Cartesian coordinates. The bottom boundary of the triangle is the line y = x+1
and the top boundary is the line y = 5 x. The x limits are 0 to 2. The integral is
_
2
0
_
5x
x+1
f(x, y) dy dx.
16.4 SOLUTIONS 1295
7. Since this is a rectangular region, we use Cartesian coordinates. The rectangle is described by the inequalities 1 x 5
and 2 y 4, so the integral is
_
5
1
_
4
2
f(x, y) dy dx.
8. This is a portion of a circle so it is best described in polar coordinates. The region is a piece of a ring in which r goes from
2 to 4. Since we include only the portion of the ring below the x-axis, we need to go from to 2. The integral is
_
2

_
4
2
f(r cos , r sin ) r dr d.
9. See Figure 16.75.
x
y
= /2
= /2

r = 4
Figure 16.75
1
1
x
y

r = 1
Figure 16.76
10. See Figure 16.76.
11. See Figure 16.77.
12. See Figure 16.78.
1 2
x
y

r = 2
%
r = 1
Figure 16.77
x
y
= /6
= /3
r = 1
Figure 16.78
x
y
= 3/4
= 3/2

r = 4

r = 3
Figure 16.79
13. See Figure 16.79.
14. See Figure 16.80.
1296 Chapter Sixteen /SOLUTIONS
x
y
= /4
r = 1/ cos
or r cos = 1
or x = 1
Figure 16.80
x
y
r = 2/ sin
or r sin = 2
or y = 2
= /4
Figure 16.81
15. See Figure 16.81.
16. By using polar coordinates, we get
_
R
sin(x
2
+y
2
)dA =
_
2
0
_
2
0
sin(r
2
)r dr d
=
_
2
0

1
2
cos(r
2
)

2
0
d
=
1
2
_
2
0
(cos 4 cos 0) d
=
1
2
(cos 4 1) 2 = (1 cos 4)
17. The region is pictured in Figure 16.82.
1 2
1
2
x
y
Figure 16.82
Using polar coordinates, we get
_
R
(x
2
y
2
)dA =
_
/2
0
_
2
1
r
2
(cos
2
sin
2
)rdr d =
_
/2
0
(cos
2
sin
2
)
1
4
r
4

2
1
d
=
15
4
_
/2
0
(cos
2
sin
2
) d
=
15
4
_
/2
0
cos 2 d
=
15
4

1
2
sin 2

/2
0
= 0.
16.4 SOLUTIONS 1297
18. The presence of the termx
2
+y
2
suggests that we should convert the integral into polar coordinates. Since
_
x
2
+y
2
= r,
the integral becomes
_
R
_
x
2
+y
2
dxdy =
_
2
0
_
3
2
r
2
drd =
_
2
0
r
3
3

3
2
d =
_
2
0
19
3
d =
38
3
.
19. (a)
x
y
1
3
y = x/3
Figure 16.83
(b)
_
1
0
_
3y
0
f(x, y) dxdy.
(c) For polar coordinates, on the line y = x/3, tan = y/x = 1/3, so = tan
1
(1/3). On the y-axis, = /2. The
quantity r goes from 0 to the line y = 1, or r sin = 1, giving r = 1/ sin and f(x, y) = f(r cos , r sin ). Thus
the integral is
_
/2
tan
1
(1/3)
_
1/ sin
0
f(r cos , r sin )r dr d.
Problems
20. By the given limits 0 x 1, and

1 x
2
y

1 x
2
, the region of integration is in Figure 16.84.
In polar coordinates, we have
_
3/2
/2
_
1
0
r cos r dr d =
_
3/2
/2
cos
_
1
3
r
3
_

1
0
d
=
1
3
_
3/2
/2
cos d
=
1
3
sin

3/2
/2
=
1
3
(1 1) =
2
3
x
y
1
Figure 16.84

6
x
y
y = x

6
y = x
Figure 16.85
21. From the given limits, the region of integration is in Figure 16.85.
In polar coordinates, /4 /4. Also,

6 = x = r cos . Hence, 0 r

6/ cos . The integral becomes


_

6
0
_
x
x
dy dx =
_
/4
/4
_

6/cos
0
r dr d
1298 Chapter Sixteen /SOLUTIONS
=
_
/4
/4
_
r
2
2

6/cos
0
_
d =
_
/4
/4
6
2 cos
2

d
= 3 tan

/4
/4
= 3 (1 (1)) = 6.
Notice that we can check this answer because the integral gives the area of the shaded triangular region which is
1
2

6
(2

6) = 6.
22. From the given limits, the region of integration is in Figure 16.86.
2
x
y

2
2

2
x
y
/4
x = y
Figure 16.86
So, in polar coordinates, we have,
_
/4
0
_
2
0
(r
2
cos sin )r dr d =
_
/4
0
cos sin
_
1
4
r
4
_

2
0
d
= 4
_
/4
0
sin(2)
2
d
= cos(2)

/4
0
= 0 (1) = 1.
23. The graph of f(x, y) = 25x
2
y
2
is an upside down bowl, and the region whose volume we want is contained between
the bowl (above) and the xy-plane (below). We must rst nd the region in the xy-plane where f(x, y) is positive. To do
that, we set f(x, y) 0 and get x
2
+y
2
25. The disk x
2
+y
2
25 is the region R over which we integrate.
Volume =
_
R
(25 x
2
y
2
) dA =
_
2
0
_
5
0
(25 r
2
) rdr d
=
_
2
0
_
25
2
r
2

1
4
r
4
_

5
0
d
=
625
4
_
2
0
d
=
625
2
24. First, lets nd where the two surfaces intersect.
_
8 x
2
y
2
=
_
x
2
+y
2
8 x
2
y
2
= x
2
+y
2
x
2
+y
2
= 4
16.4 SOLUTIONS 1299
So z = 2 at the intersection. See Figure 16.87.
x
y
z

R
2
'
'
2
x
2
+ y
2
= 4
'
Figure 16.87
The volume of the ice cream cone has two parts. The rst part (which is the volume of the cone) is the volume of the
solid bounded by the plane z = 2 and the cone z =
_
x
2
+y
2
. Hence, this volume is given by
_
R
(2
_
x
2
+y
2
) dA,
where R is the disk of radius 2 centered at the origin, in the xy-plane. Using polar coordinates, we have:
_
R
_
2
_
x
2
+y
2
_
dA =
_
2
0
_
2
0
(2 r) r dr d
=
_
2
0
_
_
r
2

r
3
3
_

2
0
_
d
=
4
3
_
2
0
d
= 8/3
The second part is the volume of the region above the plane z = 2 but inside the sphere x
2
+y
2
+z
2
= 8, which is given
by
_
R
(
_
8 x
2
y
2
2) dA where R is the same disk as before. Now
_
R
(
_
8 x
2
y
2
2) dA =
_
2
0
_
2
0
(
_
8 r
2
2)rdr d
=
_
2
0
_
2
0
r
_
8 r
2
dr d
_
2
0
_
2
0
2r dr d
=
_
2
0
_

1
3
(8 r
2
)
3/2

2
0
_
d
_
2
0
r
2

2
0
d
=
1
3
_
2
0
(4
3/2
8
3/2
) d
_
2
0
4 d
=
1
3
2(8 16

2) 8
=
2
3
(16

2 8) 8
=
8(4

2 5)
3
Thus, the total volume is the sum of the two volumes, which is 32(

2 1)/3.
1300 Chapter Sixteen /SOLUTIONS
25. The average value of the function r on the disc R of radius a is
Average of r =
1
Area of R
_
R
rdA =
1
a
2
_
2
0
_
a
0
rrdrd =
1
a
2
_
2
0
a
3
3
d =
1
a
2
2
a
3
3
=
2a
3
.
26. (a)
Total Population =
_
3/2
/2
_
4
1
(r, ) rdr d.
(b) We know that (r, ) decreases as r increases, so that eliminates (iii). We also know that (r, ) decreases as the
x-coordinate decreases, but x = r cos . With a xed r, x is proportional to cos . So as the x-coordinate decreases,
cos decreases and (i) (r, ) = (4 r)(2 + cos ) best describes this situation.
(c)
_
3/2
/2
_
4
1
(4 r)(2 + cos ) rdr d =
_
3/2
/2
(2 + cos )(2r
2

1
3
r
3
)

4
1
d
= 9
_
3/2
/2
(2 + cos ) d
= 9
_
2 + sin
_
3/2
/2
= 18( 1) 39
Thus, the population is around 39,000.
27. (a) The volume, V , is given by
V =
_
x
2
+y
2
a
2
e
(x
2
+y
2
)
dA.
Converting to polar coordinates gives
V =
_
2
0
_
a
0
e
r
2
r dr d =

2
0
_

1
2
e
r
2
_

a
0
= 2
_
1
2

1
2
e
a
2
_
= (1 e
a
2
).
(b) As a , the value of e
a
2
0, so the volume tends to .
28. The density function is given by
(r) = 10 2r
where r is the distance from the center of the disk. So the mass of the disk in grams is
_
R
(r) dA =
_
2
0
_
5
0
(10 2r)rdr d
=
_
2
0
_
5r
2

2
3
r
3
_
5
0
d
=
_
2
0
125
3
d =
250
3
(grams)
29. A rough graph of the base of the spring is in Figure 16.88, where the coil is roughly of width 0.01 inches. The volume is
equal to the product of the base area and the height. To calculate the area we use polar coordinates, taking the following
integral:
Area =
_
4
0
_
0.26+0.04
0.25+0.04
rdrd
=
1
2
_
4
0
(0.26 0.04)
2
(0.25 0.04)
2
d
16.4 SOLUTIONS 1301
=
1
2
_
4
0
0.01 (0.51 + 0.08)d
= 0.0051 2 +
1
4
(0.0008
2
)

4
0
= 0.0636
Therefore, the volume= 0.0636 0.2 = 0.0127 in
3
.
Figure 16.88
30. The charge density is = k/r, where k is a constant.
Total charge =
_
Disk
dA =
_
R
0
_
2
0
k
r
r d dr = k
_
R
0
_
2
0
d dr = k
_
R
0
2 dr = 2kR.
Thus the total charge is proportional to R with constant of proportionality 2k.
31. (a) The region (shaded) is between the circles x
2
+ y
2
= 1 and x
2
+ y
2
= 4; see Figure 16.89. The rst integral is to
the left of the dashed line x = 1; the second integral is to the right of the dashed line.
1 2
1
2
x
2
+ y
2
= 4

x
2
+ y
2
= 1
x
y
Figure 16.89
(b) Converting to polar coordinates, we nd the quantity in part (a) is given by
_
1
0
_

4x
2

1x
2
xdy dx +
_
2
1
_

4x
2
0
xdy dx =
_
/2
0
_
2
1
r cos r dr d
=
r
3
3

2
1
sin

/2
0
=
7
3
1 =
7
3
.
32. (a) The circle r = 2 cos has radius 1 and is centered at (1, 0); the circle r = 1 has radius 1 and is centered at the origin.
See Figure 16.90.
(b) The circles intersect where 2 cos = 1, so = /3. From Figure 16.90 we see that
Area =
_
/3
/3
_
2 cos
1
r dr d.
1302 Chapter Sixteen /SOLUTIONS
Evaluating gives
Area =
_
/3
/3
_
r
2
2

2 cos
1
_
d =
1
2
_
/3
/3
_
4 cos
2
1
_
d
=
1
2
(2 cos sin + 2 )

/3
/3
=
1
2
_
4
1
2

3
2
+ 2

3
_
=

3
2
+

3
.
1 2
r = 2 cos r = 1 c
= /3
T
= /3
x
y
Figure 16.90
1
2
1
x = 1/2

= /3
s
= /3
x
y
Figure 16.91
33. (a) The curve r = 1/(2 cos ) or r cos = 1/2 is the line x = 1/2. The curve r = 1 is the circle of radius 1 centered at
the origin. See Figure 16.91.
(b) The line intersects the circle where 2 cos = 1, so = /3. From Figure 16.91 we see that
Area =
_
/3
/3
_
1
1/(2 cos )
r dr d.
Evaluating gives
Area =
_
/3
/3
_
r
2
2

1
1/(2 cos )
_
d =
1
2
_
/3
/3
_
1
1
4 cos
2

_
d
=
1
2
_

tan
4
_

/3
/3
=
1
2
_
2

3
2

3
4
_
=
4 3

3
12
.
34. (a) We must rst decide where to put the origin. We locate the origin at the center of one disk and locate the center of the
second disk at the point (1, 0). See Figure 16.92. (Other choices of origin are possible.)
1
2
1

3/2

3/2

x
2
+ y
2
= 1

(x 1)
2
+ y
2
= 1
x
y
Figure 16.92
/3

r = 1

r = 2 cos
x
y
Figure 16.93
16.4 SOLUTIONS 1303
By symmetry, the points of intersection of the circles are half-way between the centers, at x = 1/2. The y-values
at these points are given by
y =
_
1 x
2
=
_
1
_
1
2
_
2
=

3
2
.
We integrate in the x-direction rst, so that it is not necessary to set up two integrals. The right-side of the circle
x
2
+y
2
= 1 is given by
x =
_
1 y
2
.
The left side of the circle (x 1)
2
+y
2
= 1 is given by
x = 1
_
1 y
2
.
Thus the area of overlap is given by
Area =
_

3/2

3/2
_

1y
2
1

1y
2
dxdy.
(b) In polar coordinates, the circle centered at the origin has equation r = 1. See Figure 16.93. The other circle, (x
1)
2
+y
2
= 1, can be written as
x
2
2x + 1 +y
2
= 1
x
2
+y
2
= 2x,
so its equation in polar coordinates is
r
2
= 2r cos ,
and, since r = 0,
r = 2 cos .
At the top point of intersection of the two circles, x = 1/2, y =

3/2, so tan =

3, giving = /3.
Figure 16.93 shows that if we integrate with respect to r rst, we have to write the integral as the sum of two
integrals. Thus, we integrate with respect to rst. To do this, we rewrite
r = 2 cos as = arccos
_
r
2
_
.
This gives the top half of the circle; the bottom half is given by
= arccos
_
r
2
_
.
Thus the area is given by
Area =
_
1
0
_
arccos(r/2)
arccos(r/2)
r ddr.
35. (a) The density increases at a rate of (25 1)/12 = 2 gm/cm
2
for each cm of radius. Thus, at radius r,
Density = 1 + 2r gm/cm
2
.
Thus
Mass =
_
2
0
_
12
0
(1 + 2r) r dr d gm.
(b) Evaluating gives
Mass =
_
2
0
_
r
2
2
+
2
3
r
3
_

12
0
d = 2 1224 = 2448 gm.
1304 Chapter Sixteen /SOLUTIONS
Solutions for Section 16.5
Exercises
1. (a) is (IV); (b) is (II); (c) is (VII); (d) is (VI); (e) is (III); (f) is (V).
2. The cone has equation z = r.
3. The cone has equation = /4.
4. The top half of the sphere has equation z =
_
1 x
2
y
2
=

1 r
2
.
5. The plane has equation = /4.
6. The plane has equation cos = 4 or = 4/ cos .
7. The plane has equation cos = 10 or = 10/ cos .
8.
_
W
f dV =
_
1
1
_
3/4
/4
_
4
0
(r
2
+z
2
) rdr d dz
=
_
1
1
_
3/4
/4
(64 + 8z
2
) d dz
=
_
1
1

2
(64 + 8z
2
) dz
= 64 +
8
3
=
200
3

9.
_
W
f dV =
_
3
1
_
2
0
_
1
0
(sin (r
2
)) rdr d dz
=
_
3
1
_
2
0
(
1
2
cos r
2
)

1
0
d dz
=
1
2
_
3
1
_
2
0
(cos 1 cos 0) d dz
=
_
3
1
(cos 1 1) dz = 4(cos 1 1) = 4(1 cos 1)
10. We have:
_
W
f dV =
_
5
0
_
2
0
_

/2
1


2
sin dd d
=
_
5
0
_
2
0
_

/2
sin dd d
=
_
5
0
_
2
0
d d
= 2
_
5
0
d = 25
Note that the integral is improper, but it can be shown that the result is correct.
11.
_
W
f dV =
_
2
0
_
/4
0
_
2
1
(sin )
2
sin d dd
16.5 SOLUTIONS 1305
=
_
2
0
_
/4
0
_
2
1

2
sin
2
d dd
=
7
3
_
2
0
_
4
0
sin
2
dd
=
7
3
_
2
0
_
/4
0
1 cos 2
2
dd
=
7
6
_
2
0
(
1
2
sin 2)

/4
0
d
=
7
6
_
2
0
(

4

1
2
) d
=
7
6
2(

4

1
2
) =
7( 2)
12
12. Using Cartesian coordinates, we get:
_
3
0
_
1
0
_
5
0
f dz dy dx
13. Using cylindrical coordinates, we get:
_
1
0
_
2
0
_
4
0
f rdr d dz
14. Using cylindrical coordinates, we get:
_
4
0
_
/2
0
_
2
0
f rdr d dz
15. Using spherical coordinates, we get:
_

0
_

0
_
3
2
f
2
sin d dd
16. Using spherical coordinates, we get:
_
2
0
_
/6
0
_
3
0
f
2
sin d dd
17. We use Cartesian coordinates, oriented as shown in Figure 16.94. The slanted top has equation z = mx, where m is the
slope in the x-direction, so m = 1/5. Then if f is an arbitrary function, the triple integral is
_
5
0
_
2
0
_
x/5
0
f dzdydx.
Other answers are possible.
x
y
z
1
2
5
(5, 2, 0)
(5, 2, 1)
c
(5, 0, 1)
Figure 16.94
1306 Chapter Sixteen /SOLUTIONS
18. We choose cylindrical coordinates oriented as in Figure 16.95. The cone has equation z = r. Since we have a half cone
scooped out of a half cylinder, varies between 0 and . Thus, if f is an arbitrary function, the integral is
_

0
_
2
0
_
r
0
fr dzdrd.
Other answers are possible.
2
2
z = r
/4
x
z
Figure 16.95
x
y
z
Figure 16.96
19. R is one eighth of a sphere of radius 1, below the xy-plane and under the rst quadrant. See Figure 16.96.
20. (a) The region of integration is the region between the cone z = r, the xy-plane and the cylinder r = 3. In spherical
coordinates, r = 3 becomes sin = 3, so = 3/ sin . The cone is = /4 and the xy-plane is = /2. See
Figure 16.97. Thus, the integral becomes
_
2
0
_
/2
/4
_
3/ sin
0

2
sin ddd.
x
z
3
r = 3
z = r

4
Figure 16.97: Region of integration is
between the cone and the xy-plane
(b) The original integral is easier to evaluate, so
_
2
0
_
3
0
_
r
0
r dzdrd =
_
2
0
_
3
0
zr

z=r
z=0
drd =
_
2
0
_
3
0
r
2
drd = 2
r
3
3

3
0
= 18.
Problems
21. We want the volume of the region above the cone = /3 and below the sphere = 3:
V =
_
2
0
_
/3
0
_
3
0

2
sin d dd.
The order of integration can be altered and other coordinates can be used.
16.5 SOLUTIONS 1307
22. We want the volume of the region between the sphere = 3 and the cone z = r. The sphere can also be written
x
2
+y
2
+z
2
= 3
2
or r
2
+z
2
= 9. The cone can also be written as = /4.
The sphere cuts the cone z = r in the circle 2r
2
= 9, or r = 3/

2, lying in the plane z = 3/

2.
In cylindrical coordinates,
V =
_
2
0
_
3/

2
0
_

9r
2
r
r dz dr d.
In spherical coordinates
V =
_
2
0
_
/4
0
_
3
0

2
sin d dd.
The order of integration can be altered and other coordinates can be used.
23. The cone can be written z = r, and the rst quadrant of the xy-plane is given by 0 /2. The region x
2
+y
2
7
is given by r

7. Thus
V =
_
/2
0
_

7
0
_
r
0
r dz dr d.
The order of integration can be altered and other coordinates can be used.
24. In cylindrical coordinates, the region is the half cylinder given by 5 z 10,

2 r

3, 0 . Thus
V =
_

0
_

3

2
_
10
5
r dz dr d.
The order of integration can be altered and other coordinates can be used.
25. We use cylindrical coordinates since the sphere x
2
+ y
2
+ z
2
= 10, or r
2
+ z
2
= 10, and the plane z = 1 can both be
simply expressed. The plane cuts the sphere in the circle r
2
+ 1
2
= 10, or r = 3. Thus
V =
_
2
0
_
3
0
_

10r
2
1
r dz dr d,
or
V =
_
2
0
_

10
1
_

10z
2
0
r dr dz d.
Order of integration can be altered and other coordinates can be used.
26. In spherical coordinates, the cone z = r is given by = /4 and the sphere x
2
+ y
2
+ z
2
= 8 is given by =

8.
Since is measured from the positive z-axis, the region we are interested in has /4 /2. Thus
V =
_
2
0
_
/2
/4
_

8
0

2
sin d dd.
The order of integration can be altered and other coordinates can be used.
27. We use cylindrical coordinates. The cone has radius r = 2 when z = 4, so its equation is z = 2r. Thus, the integral is
_
2
0
_
2
0
_
4
2r
f(r, , z)r dz dr d.
28. We use spherical coordinates. The cone has radius 2 when z = 4, so sin = 2 when cos = 4. Thus tan = 1/2,
so = arctan(1/2). The top of the cone, z = 4, is given by cos = 4. Thus, the integral is
_
2
0
_
arctan(1/2)
0
_
4/ cos
0
g(, , )
2
sin d dd.
29. In rectangular coordinates, a cone has equation z = k
_
x
2
+y
2
for some constant k. Since z = 4 when
_
x
2
+y
2
=

2
2
= 2, we have k = 2. Thus, the integral is
_
2
2
_

4x
2

4x
2
_
4
2

x
2
+y
2
h(x, y, z) dz dy dx.
1308 Chapter Sixteen /SOLUTIONS
30. (a) In Cartesian coordinates, the bottom half of the sphere x
2
+y
2
+z
2
= 1 is given by z =
_
1 x
2
y
2
. Thus
_
W
dV =
_
1
0
_

1x
2
0
_
0

1x
2
y
2
dz dy dx.
(b) In cylindrical coordinates, the sphere is r
2
+z
2
= 1 and the bottom half is given by z =

1 r
2
. Thus
_
W
dV =
_
/2
0
_
1
0
_
0

1r
2
r dz dr d.
(c) In spherical coordinates, the sphere is = 1. Thus,
_
W
dV =
_
/2
0
_

/2
_
1
0

2
sin d dd.
31. (a) Since the cone has a right angle at its vertex, it has equation
z =
_
x
2
+y
2
.
The sphere has equation x
2
+y
2
+z
2
= 1, so the top half is given by
z =
_
1 x
2
y
2
.
The cone and the sphere intersect in the circle
x
2
+y
2
=
1
2
, z =
1

2
.
See Figure 16.98. Thus
_
W
dV =
_
1/

2
1/

2
_

(1/2)x
2

(1/2)x
2
_

1x
2
y
2

x
2
+y
2
dz dy dx.
x
y
z
1
s
x
2
+ y
2
=
1
2
s
Circle is
x
2
+ y
2
=
1
2
T
c
1

2
4
Figure 16.98
(b) In cylindrical coordinates, the cone has equation z = r and the sphere has equation z =

1 r
2
. Thus
_
W
dV =
_
2
0
_
1/

2
0
_

1r
2
r
r dz dr d.
(c) In spherical coordinates, the cone has equation = /4 and the sphere is = 1. Thus
_
W
dV =
_
2
0
_
/4
0
_
1
0

2
sin d dd.
16.5 SOLUTIONS 1309
32. (a) Since the cone has a right angle at its vertex, it has equation
z =
_
x
2
+y
2
.
Figure 16.99 shows the plane with equation z = 1/

2. The plane and the cone intersect in the circle x


2
+y
2
= 1/2.
Thus,
_
W
dV =
_
1/

2
1/

2
_

(1/2)x
2

(1/2)x
2
_
1/

x
2
+y
2
dz dy dx.
x
y
z
1

2
1

4
1
Figure 16.99
(b) In cylindrical coordinates the cone has equation z = r, so
_
W
dV =
_
2
0
_
1/

2
0
_
1/

2
r
r dz dr d.
(c) In spherical coordinates, the cone has equation = /4 and the plane z = 1/

2 has equation cos = 1/

2.
Thus
_
W
dV =
_
2
0
_
/4
0
_
1/(

2 cos )
0

2
sin d dd.
33. The region is a solid cylinder of height 1, radius 1 with base on the xy-plane and axis on the z-axis. We have:
_
1
0
_
1
1
_

1x
2

1x
2
1
(x
2
+y
2
)
1/2
dy dxdz =
_
1
0
_
2
0
_
1
0
1
r
r dr d dz
=
_
1
0
_
2
0
r

1
0
d dz
=
_
1
0
_
2
0
d dz = 2.
Note that the integral is improper, but it can be shown that the result is correct.
34. The region of integration is half of a ball centered at the origin, radius 1, on the x 0 side. Since the integral is symmetric,
we can integrate over the quarter unit ball (x 0, y 0) and multiply the result by 2. We have:
_
1
0
_

1x
2

1x
2
_

1x
2
z
2

1x
2
z
2
1
(x
2
+y
2
+z
2
)
1/2
dy dz dx
= 2
_
/2
0
_

0
_
1
0
1

2
sin d dd
= 2
_
/2
0
_

0

2
2

1
0
sin dd
=
_
/2
0
(cos )

0
d
= ((1) 0) = .
Note that the integral is improper, but it can be shown the result is correct.
1310 Chapter Sixteen /SOLUTIONS
35. (a) The region is the half cylinder in Figure 16.100.
1
1
1
x
y
z
Figure 16.100
(b) Use cylindrical coordinates with x replacing z and y in place of x and z in place of y. Then
_
1
1
_
1
1
_

1z
2
0
f(x, y, z) dy dz dx =
_
1
1
_
/2
/2
_
1
0
r
3
r dr d dx = x

1
1

/2
/2
r
5
5

1
0
=
2
5
.
36. For x
2
+ y
2
1, the cone is below the plane z = 10 + x. In cylindrical coordinates, the plane is z = 10 + r cos , and
the cone is z = r. Thus
Volume =
_
2
0
_
1
0
_
10+r cos
r
r dz dr d
=
_
2
0
_
1
0
rz

10+r cos
r
dr d
=
_
2
0
_
1
0
(10r +r
2
cos r
2
) dr d
=
_
2
0
_
5r
2
+
r
3
3
cos
r
3
3
_

1
0
d
=
_
2
0
_
5 +
1
3
cos
1
3
_
d
=
_
14
3
+
1
3
sin
_

2
0
=
28
3
.
37. The cone is centered along the positive x-axis and intersects the sphere in the circle
(y
2
+z
2
) +y
2
+z
2
= 4
y
2
+z
2
= 2.
We use spherical coordinates with measured from the x-axis and measured in the yz-plane. (Alternatively, the volume
we want is equal to the volume between the cone z =
_
x
2
+y
2
and the sphere x
2
+ y
2
+ z
2
= 4.) The cone is given
by = /4. The sphere has equation = 2. Thus
Volume =
_
2
0
_
/4
0
_
2
0

2
sin d dd
=
_
2
0
_
/4
0

3
3
sin

2
0
dd
16.5 SOLUTIONS 1311
=
_
2
0
_
/4
0
8
3
sin dd
=
_
2
0

8
3
cos

/4
0
d =
_
2
0
8
3
_
1
1

2
_
d
=
16
3
_
1
1

2
_
.
38. Using cylindrical coordinates, the equation of the sphere is r
2
+z
2
= 4. The top of the sphere has equation z =

4 r
2
.
When z = 1 we have r =

3. Figure 16.101 shows the limits of integration on the integral.


Volume =
_
2
0
_

3
0
_

4r
2
1
r dz dr d
= 2
_

3
0
rz

4r
2
1
dr = 2
_

3
0
(r
_
4 r
2
r) dr
= 2
_
(4 r
2
)
3/2
3

r
2
2
_

3
0
=
5
3
.
1
2
/3
/6

3
T
c
1
r
z
Figure 16.101
5/

2
A B C
O
)
/4

/4
5
r
z
Figure 16.102
39. Orient the cone as shown in Figure 16.102 and use cylindrical coordinates with the origin at the vertex of the cone. Since
the angle at the vertex of the cone is a right angle, the angles AOB and COB are both /4. Thus, OB = 5 cos /4 =
5/

2. The curved surface of the cone has equation z = r, so


Volume =
_
2
0
_
5/

2
0
_
5/

2
r
r dz dr d
=
_
2
0
_
5/

2
0
rz

z=5/

2
z=r
dr d =
_
2
0
_
5/

2
0
r
_
5

2
r
_
dr d
=

2
0
_
5

2
r
2
2

r
3
3
_

5/

2
0
= 2
_
5

5
2
2
2

5
3
2 3

2
_
= 2
5
3
2

2
_
1
2

1
3
_
=
5
3

2
= 46.28 cm
3
.
40. Use cylindrical coordinates: when r
2
= x
2
+ y
2
= 1, then x
2
+ y
2
+ z
2
= 1 + z
2
= 2 so z = 1. The region W is
shown in Figure 16.103.
_
W
(x
2
+y
2
) dV =
_
1
1
_
2
0
_

2z
2
1
r
2
r drddz
=
_
1
1
_
2
0
r
4
4

2z
2
1
ddz =
1
4
_
1
1
_
2
0
_
(2 z
2
)
2
1
_
ddz
1312 Chapter Sixteen /SOLUTIONS
=
2
4
_
1
1
_
3 4z
2
+z
4
_
dz
=

2
_
3z
4
3
z
3
+
z
5
5
_

1
1
=
28
15
.
x
y
z

Cylinder
x
2
+ y
2
= 1

Sphere
x
2
+ y
2
+ z
2
= 2
1
1
Figure 16.103
x
y
z
=

6
=

3
5
T
c
2
Figure 16.104
41. The region whose volume we want is shown in Figure 16.104: Using cylindrical coordinates, the volume is given by the
integral:
V =
_
2
0
_
/3
/6
_
5
0
r dr d dz
=
_
2
0
_
/3
/6
r
2
2

5
0
d dz
=
25
2
_
2
0
_
/3
/6
d dz
=
25
2
_
2
0
_

3


6
_
dz
=
25
2


6
2 =
25
6
.
42. (a) The angle takes on values in the range 0 . Thus, sin is nonnegative everywhere in W1, and so its integral
is positive.
(b) The function is symmetric across the xy plane, such that for any point (x, y, z) in W1, with z = 0, the point
(x, y, z) has a cos value with the same magnitude but opposite sign of the cos value for (x, y, z). Furthermore,
if z = 0, then (x, y, z) has a cos value of 0. Thus, with cos positive on the top half of the sphere and negative on
the bottom half, the integral will cancel out and be equal to zero.
43. (a) The integral is negative. In W2, we have 0 < z < 1. Thus, z
2
z is negative throughout W2 and thus its integral is
negative.
(b) On the top half of the sphere, z is nonnegative, but x can be both positive and negative. Thus, since W2 is symmetric
with respect to the yz plane, the contribution of a point (x, y, z) will be canceled out by its reection (x, y, z).
Thus, the integral is zero.
44. The surface z = 4 x
2
y
2
cuts the xy-plane in the circle x
2
+y
2
= 4. Thus
Mass =
_
2
2
_

4x
2

4x
2
_
4x
2
y
2
0
e
xy
dz dy dx gm.
16.5 SOLUTIONS 1313
45. The cylinder has radius 2. Using cylindrical coordinates to nd the mass and integrating with respect to r rst, we have
Mass =
_
2
0
_
3
0
_
2
0
(1 +r)r dr dz d =
_
2
0
_
3
0
_
r
2
2
+
r
3
3
_

2
0
dz d = 2 3
_
4
2
+
8
3
_
= 28 gm.
46. In cylindrical coordinates, the density, is given by = kr for some positive constant k.
For the smaller cylinder, x
2
+y
2
1, 0 z 2, whose radius is 1,
Mass =
_
2
0
_
2
0
_
1
0
kr r dr dz d = 2 2
kr
3
3

1
0
=
4k
3
.
For the larger cylinder, x
2
+y
2
9, 0 z 2, whose radius is 3,
Mass =
_
2
0
_
2
0
_
3
0
kr r dr dz d = 2 2
kr
3
3

3
0
= 36k.
Thus, the ratio of the masses is
4k/3
36k
=
1
27
.
47. Using cylindrical coordinates, the density is given by = kr
2
gm/cm
3
, where k is a constant. Since = 2 when r = 2,
we have
2 = k2
2
so k = 0.5.
The equation of the sphere is x
2
+y
2
+z
2
= 3
2
, and in cylindrical coordinates,
r
2
+z
2
= 9.
Thus r =

9 z
2
on the sphere, so
Mass =
_
2
0
_
3
3
_

9z
2
0
(0.5r
2
)r dr dz d
=
_
2
0
_
3
3
0.5
r
4
4

9z
2
0
dz d
=
_
2
0
_
3
3
1
8
(9 z
2
)
2
dz d
=
1
8
_
2
0
_
3
3
(81 18z
2
+z
4
) dz d
=
2
8
_
81z
18z
3
3
+
z
5
5
_

3
3
=
324
5
gm.
48. We use spherical coordinates. The density, , of the sphere at a distance from the center is
= k
2
for k a positive constant.
Thus, for a sphere of radius 1,
Mass =
_
2
0
_

0
_
1
0
k
2

2
sin d dd
= 2
_

0
k

5
5
sin

1
0
d =
2k
5
(cos )

0
=
4k
5
.
1314 Chapter Sixteen /SOLUTIONS
For a sphere of radius 2, a similar calculation gives
Mass =
_
2
0
_

0
_
2
0
k
2

2
sin d dd
= 2k
2
5
5
(cos )

0
=
128k
5
.
Therefore
Ratio of masses =
4k/5
128k/5
=
1
32
.
49. (a) We use spherical coordinates. Since = 9 where = 6 and = 11 where = 7, the density increases at a rate
of 2 gm/cm
3
for each cm increase in radius. Thus, since density is a linear function of radius, the slope of the linear
function is 2. Its equation is
11 = 2( 7) so = 2 3.
(b) Thus,
Mass =
_
2
0
_

0
_
7
6
(2 3)
2
sin d dd.
(c) Evaluating the integral, we have
Mass = 2
_
cos

0
_
_
2
4
4

3
3
3

7
6
_
= 2 2(425.5) = 1702 gm = 5346.991 gm.
50. The distance from a point (x, y, z) to the origin is given by
_
x
2
+y
2
+z
2
. Thus we want to evaluate
_
R
_
x
2
+y
2
+z
2
dV
Vol(R)
where R is the region bounded by the hemisphere z =
_
8 x
2
y
2
and the cone z =
_
x
2
+y
2
. See Figure 16.105.
We will use spherical coordinates.
x
y
z
'
'
2
x
2
+ y
2
= 4
'
Figure 16.105
16.5 SOLUTIONS 1315
In spherical coordinates, the quantity goes from 0 to

8, and goes from 0 to 2, and goes from 0 to /4
(because the angle of the cone is /4). Thus we have
_
R
_
x
2
+y
2
+z
2
dV =
_
2
0
_
/4
0
_

8
0
(
2
sin ) ddd
=
_
2
0
_
/4
0
sin

4
4

8
0
dd
=
_
2
0
_
/4
0
16 sin dd
=
_
2
0
16(cos )

/4
0
d
=
_
2
0
16
_
1

2
2
_
d
= 32
_
1

2
2
_

From Problem 24 of Section 16.4 we know that Vol(R) = 32(

2 1)/3, therefore
Average distance =
_
R
_
x
2
+y
2
+z
2
dV
Vol(R)
=
32
_
1

2
2
_

[32(

2 1)/3]
=
3

2
.
51. (a) First we must choose a coordinate system, since none is given. We pick the xy-plane to be the xed plane and the
z-axis to be the line perpendicular to the plane. Then the distance from a point to the plane is |z|, so the density at a
point is given by
Density = = k|z|.
Using cylindrical coordinates for the integral, we nd
Mass =
_
2
0
_
a
0
_

a
2
r
2

a
2
r
2
k|z|r dzdrd.
(b) By symmetry, we can evaluate this integral over the top half of the sphere, where |z| = z. Then
Mass = 2
_
2
0
_
a
0
_

a
2
r
2
0
kzr dzdrd = 2k
_
2
0
_
a
0
z
2
2
r

z=

a
2
r
2
z=0
drd
= k
_
2
0
_
a
0
r(a
2
r
2
) drd = k2
_
r
2
2
a
2

r
4
4
_

a
0
= 2k
_
a
4
2

a
4
4
_
=
ka
4
2
.
52. (a) We use the axes shown in Figure 16.106. Then the sphere is given by r
2
+z
2
= 25, so
Volume =
_
2
0
_
5
1
_

25r
2

25r
2
r dzdrd.
(b) Evaluating gives
Volume = 2
_
5
1
rz

z=

25r
2
z=

25r
2
dr = 2
_
5
1
2r
_
25 r
2
dr
1316 Chapter Sixteen /SOLUTIONS
= 2
_

2
3
_
(25 r
2
)
3/2

5
1
=
4
3
(24)
3/2
= 64

6 = 492.5 mm
3
.
1 5
5

r
2
+ z
2
= 25
x
2
+ y
2
+ z
2
= 25
x
y
Figure 16.106
53. We must rst decide on coordinates. We pick spherical coordinates with the common center of the two spheres as the
origin. We imagine the half-melon with the at side horizontal and the positive z-axis going through the curved surface.
See Figure 16.107. The volume is given by the integral
Volume =
_
2
0
_
/2
0
_
b
a

2
sin ddd.
Evaluating gives
Volume =
_
2
0
_
/2
0
sin

3
3

p=b
p=a
dd = 2(cos )

/2
0
_
b
3
3

a
3
3
_
=
2
3
(b
3
a
3
).
To check our answer, notice that the volume is the difference between the volumes of two half spheres of radius a and b.
These half spheres have volumes 2b
3
/3 and 2a
3
/3, respectively.
x
y
z
a
b
Figure 16.107
54. Integrating with respect to z rst, we have
W =
_
1
0
_
2
0
_
(

9r
2
)1

1r
2
r dz ddr +
_
2

2
1
_
2
0
_
(

9r
2
)1
0
r dz d dr
or integrating with respect to r rst, we have
_
1
0
_
2
0
_

9(z+1)
2

1z
2
r dr d dz +
_
2
1
_
2
0
_

9(z+1)
2
0
r dr d dz.
16.5 SOLUTIONS 1317
55. The total volume of the cone is
1
3
r
2
h =
1
3
1
2
1 =
1
3
, so the total mass is
1
3
(since the density is always 1). The
center of mass z-coordinate is given by
z =
3

_
C
z dV
Using cylindrical coordinates to evaluate this integral gives
z =
3

_
2
0
_
1
0
_
z
0
zr dr dz d
=
3

_
2
0
_
1
0
z
3
2
dz d
=
3

_
2
0
1
8
d =
3
4
56. (a) The mass m of the cone is given by
_
C
dV . In cylindrical coordinates this is
m =
_
2
0
_
1
0
_
z
0
z
2
r dr dz d
=
_
2
0
_
1
0
z
4
2
dz d
=
_
2
0
1
10
d =

5
(b) The center of mass z-coordinate is given by
z =
5

_
C
z z
2
dV
Using cylindrical coordinates to evaluate this integral gives
z =
5

_
2
0
_
1
0
_
z
0
z
3
r dr dz d
=
5

_
2
0
_
1
0
z
5
2
dz d
=
5

_
2
0
1
12
d =
5
6
Comparing this answer with the center of mass in Problem 55, where the density was constant, it makes sense
that the center of mass would be higher in this problem, since more mass is concentrated near the top of the cone.
57. We rst need to nd the mass of the solid, using cylindrical coordinates:
m =
_
2
0
_
1
0
_

z/a
0
r dr dz d
=
_
2
0
_
1
0
z
2a
dz d
=
_
2
0
1
4a
d =

2a
It makes sense that the mass would vary inversely with a, since increasing a makes the paraboloid skinnier. Now for
the z-coordinate of the center of mass, again using cylindrical coordinates:
z =
2a

_
2
0
_
1
0
_

z/a
0
zr dr dz d
1318 Chapter Sixteen /SOLUTIONS
=
2a

_
2
0
_
1
0
z
2
2a
dz d
=
2a

_
2
0
1
6a
d =
2
3
58. The volume of the hemisphere is
2
3
a
3
so its mass is
2
3
a
3
b. To nd the location of the center of mass, we place the
base of the hemisphere on the xy-plane with the origin at its center, so we can describe it in spherical coordinates by
0 a, 0

2
and 0 2. Then the x-coordinate of the center of mass is, integrating using spherical
coordinates:
x =
3
2a
3
b
_
a
0
_
2
0
_
2
0
sin() cos()
2
sin() d dd = 0
since the rst integral
_
2
0
cos() d is zero. A similar computation shows that y = 0. Now for the z-coordinate:
z =
3
2a
3
b
_
a
0
_
2
0
_
2
0
cos()
2
sin() d dd
=
3
2a
3
b
2
_
a
0
_
2
0

3
cos() sin() dd
=
3
a
3
b
_
a
0

3
sin
2
()
2

2
0
d
=
3
2a
3
b
_
a
0

3
d =
3a
8b
So the x and y-coordinates are located at the center of the base, while the z-coordinate is located
3a
8b
above the center of
the base.
59. The sum of the three moments of inertia I for the ball B will be
3I =
3
4a
3
_
B
(y
2
+z
2
) dV +
3
4a
3
_
B
(x
2
+z
2
) dV +
3
4a
3
_
B
(x
2
+y
2
) dV
=
3
4a
3
_
B
(2x
2
+ 2y
2
+ 2z
2
) dV,
which, in spherical coordinates is
3
2a
3
_
B
(x
2
+y
2
+z
2
) dV =
3
2a
3
_
a
0
_

0
_
2
0

2

2
sin() d dd
=
3
a
3
_
a
0
_

0

4
sin() dd
=
6
a
3
_
a
0

4
d =
6
5
a
2
.
Thus 3I =
6
5
a
2
, so I =
2
5
a
2
.
60. First we need to nd the volume of the cone. In spherical coordinates we nd:
V =
_
a
0
_
3
0
_
2
0

2
sin() d dd =
a
3
3
Now, to nd the moment of inertia about the z-axis we need to compute the integral
3
a
3
_
W
x
2
+y
2
dV . We can do
this in spherical coordinates as
3
a
3
_
W
x
2
+y
2
dV =
3
a
3
_
a
0
_
3
0
_
2
0
(
2
sin
2
() cos
2
() +
2
sin
2
() sin
2
())
2
sin() d dd
16.5 SOLUTIONS 1319
=
3
a
3
_
a
0
_
3
0
_
2
0

4
sin
3
() d dd
=
6
a
3
_
a
0
_
3
0

4
sin
3
() dd
=
6
a
3
5
24
_
a
0

4
d =
a
2
4
.
61. Assume the base of the cylinder sits on the xy-plane with center at the origin. Because the cylinder is symmetric about the
z-axis, the force in the horizontal x or y direction is 0. Thus we need only compute the vertical z component of the force.
We are going to use cylindrical coordinates; since the force is G mass/(distance)
2
, a piece of the cylinder of volume
dV located at (r, , z) exerts on the unit mass a force with magnitude G( dV )/(r
2
+z
2
). See Figure 16.108.
Vertical component
of force
=
G( dV )
r
2
+z
2
cos =
G dV
r
2
+z
2

z

r
2
+z
2
=
Gz dV
(r
2
+z
2
)
3/2
.
Adding up all the contributions of all the dV s, we obtain
Vertical force =
_
H
0
_
2
0
_
R
0
Gzr
(r
2
+z
2
)
3/2
drddz
=
_
H
0
_
2
0
(Gz)
_

r
2
+z
2
_

R
0
ddz
=
_
H
0
_
2
0
(Gz)
_

R
2
+z
2
+
1
z
_
ddz
=
_
H
0
2G
_
1
z

R
2
+z
2
_
dz
= 2G(z
_
R
2
+z
2
)

H
0
= 2G(H
_
R
2
+H
2
+R) = 2G(H +R
_
R
2
+H
2
)

r
2
+ z
2
z

Figure 16.108
62. The charge density is = kz, where k is a constant. In cylindrical coordinates,
Total charge =
_
Cylinder
dV =
_
h
0
_
R
0
_
2
0
kzr d dr dz = k
_
h
0
_
R
0
2zr dr dz
= k
_
h
0
R
2
z dz = k(R
2
)
h
2
2
=
k
2
R
2
h
2
.
Thus, the total charge is proportional to R
2
h
2
with constant of proportionality k/2.
1320 Chapter Sixteen /SOLUTIONS
63. The charge density is = k/. Integrating in spherical coordinates,
Total charge =
_
2
0
_

0
_
R
0
k

2
sin d dd = k
_
2
0
_

0
R
2
2
sin dd
= 4k
R
2
2
= 2kR
2
.
Thus, the total charge is proportional to R
2
with constant of proportionality 2k.
64. Using spherical coordinates,
Stored energy =
1
2
_
b
a
_

0
_
2
0
E
2

2
sin d dd =
q
2
32
2

_
b
a
_

0
_
2
0
1

2
sin d dd
=
q
2
8
_
b
a
1

2
d =
q
2
8
_
1
a

1
b
_
.
65. Use cylindrical coordinates, with the z-axis being the axis of the cable. Consider a piece of cable of length 1. Then
Stored energy =
1
2
_
b
a
_
1
0
_
2
0
E
2
r d dz dr =
q
2
8
2

_
b
a
_
1
0
_
2
0
1
r
d dz dr
=
q
2
4
_
b
a
1
r
dr =
q
2
4
(ln b ln a) =
q
2
4
ln
b
a
.
So the stored energy is proportional to ln(b/a) with constant of proportionality q
2
/4.
66. In the system used in this book the volume element is dV =
2
sin d dd. In the system shown in the problem,
and have been interchanged and changed to r. So the volume element is dV = r
2
sin dr d d.
Solutions for Section 16.6
Exercises
1. We have p(x, y) = 0 for all points (x, y) satisfying x 3, since all such points lie outside the region R. Therefore the
fraction of the population satisfying x 3 is 0.
2. The fraction is 0, since
_
1
1
xy dx = 0, so
_

_
1
1
p(x, y) dxdy =
_
1
0
_
1
1
xy dxdy=0.
3. Since x +y 3 for all points (x, y) in the region R, the fraction of the population satisfying x +y 3 is 1.
4. Since p(x, y) = 0 for any (x, y) with x < 0 and also p(x, y) = 0 for any (x, y) with y > 1 or y < 0, the fraction of the
population is given by the double integral:
_
1
0
_
1
0
xy dxdy =
_
1
0
x
2
y
2

1
0
dy =
_
1
0
y
2
dy =
y
2
4

1
0
=
1
4
.
5. Since p(x, y) = 0 for all (x, y) outside the rectangle R, the population is given by the volume under the graph of p over
the region inside the rectangle R and to the right of the line x = y. Therefore the fraction of the population is given by
the double integral:
_
1
0
_
2
y
xy dxdy =
_
1
0
x
2
y
2

2
y
dy =
_
1
0
_
2y
y
3
2
_
dy =
_
y
2

y
4
8
_

1
0
=
7
8
.
16.6 SOLUTIONS 1321
6. Since p(x, y) = 0 for all (x, y) outside the rectangle R, the population is given by the volume under the graph of p over
the region inside the rectangle R and below the line x +y = 1. This is the same as the region bounded by the x-axis, the
y-axis, and the line x +y = 1. Therefore the fraction of the population is given by the double integral:
_
1
0
_
1y
0
xy dxdy =
_
1
0
x
2
y
2

1y
0
dy =
_
1
0
(1 y)
2
y
2
dy =
_
y
2
4

y
3
3
+
y
4
8
_

1
0
=
1
24
.
7. The fraction of the population is given by the double integral:
_
1/2
0
_
1
0
xy dxdy =
_
1/2
0
x
2
y
2

1
0
dy =
_
1/2
0
y
2
dy =
y
2
4

1/2
0
=
1
16
.
8. We are looking for points inside the circle x
2
+ y
2
= 1 and inside the rectangle R. In the rst quadrant, all of the circle
and its interior lies inside the rectangle R. Thus the fraction of the population we want is given by the volume under the
graph of p over the region inside the circle x
2
+y
2
= 1 in the rst quadrant. We evaluate this double integral using polar
coordinates:
_
/2
0
_
1
0
(r cos )(r sin ) r dr d =
_
/2
0
r
4
4
cos sin

1
0
d =
1
4
_
/2
0
cos sin d.
Making the substitution w = sin , we get:
_
/2
0
cos sin d =
_
1
0
wdw =
1
2
.
Thus the fraction is (1/4)(1/2) = 1/8.
9. No, p is not a joint density function. Since p(x, y) = 0 outside the region R, the volume under the graph of p is the same
as the volume under the graph of p over the region R, which is 2 not 1.
10. Yes, p is a joint density function. The values of p(x, y) are nonnegative, since p(x, y) = 1/2 for all points inside R and
p(x, y) = 0 for all other points. The volume under the graph of p over the region R is (1/2)(5 4)(0 (2)) = 1.
11. No, p is a not joint density function, because p(x, y) < 0 for some points (x, y) in the region R. For example,
p(0.7, 0.1) = 0.6.
12. Yes, p is a joint density function. Since x y everywhere in the region R, we have p(x, y) = 6(y x) 0 for all x
and y in R, and p(x, y) = 0 for all other (x, y). To check that p is a joint density function, we check that the total volume
under the graph of p over the region R is 1:
_
R
p(x, y) dA =
_
1
0
_
y
0
6(y x) dxdy =
_
1
0
6
_
yx
x
2
2
_

y
0
dy =
_
1
0
3y
2
dx = y
3

1
0
= 1.
13. Yes, p is a joint density function. In the region R we have 1 x
2
+ y
2
, so p(x, y) = (2/)(1 x
2
y
2
) 0 for all x
and y in R, and p(x, y) = 0 for all other (x, y). To check that p is a joint density function, we check that the total volume
under the graph of p over the region R is 1. Using polar coordinates, we get:
_
R
p(x, y)dA =
2

_
2
0
_
1
0
(1 r
2
)r dr d =
2

_
2
0
_
r
2
2

r
4
4
_

1
0
d =
2

_
2
0
1
4
d = 1.
14. Yes, p is a joint density function. Since e
xy
is always positive, p(x, y) = xye
xy
0 for all x and y in R, and
hence for all x and y. To check that p is a joint density function, we check that the total volume under the graph of p over
the region R is 1. Since e
xy
= e
x
e
y
, we have
_
R
xye
xy
dA =
_

0
_

0
xye
xy
dxdy =
_

0
ye
y
__

0
xe
x
dx
_
dy.
Using integration by parts:
_

0
xe
x
dx = lim
b
(xe
x
e
x
)

b
0
= (0 0) (0 1) = 1.
1322 Chapter Sixteen /SOLUTIONS
Thus
_
R
xye
xy
dA =
_

0
ye
y
__

0
xe
x
dx
_
dy =
_

0
ye
y
dy = 1.
15. (a)
_
1
0
_
1
1/3
2
3
(x + 2y) dxdy =
_
1
0
2
3
(
1
2
x
2
+ 2xy)

1
1/3
dy
=
_
1
0
2
3
_
(
1
2
+ 2y) (
1
18
+
2
3
y)
_
dy
=
2
3
_
1
0
(
4
9
+
4
3
y) dy
=
2
3
_
4
9
y +
2
3
y
2
_

1
0
=
2
3
_
10
9
_
=
20
27
.
(b) It is easier to calculate the probability that x < (1/3)+y does not happen, that is, the probability that x (1/3)+y,
and subtract it from 1. The probability that x (1/3) +y is
_
1
1/3
_
x(1/3)
0
2
3
(x + 2y) dy dx =
_
1
1/3
2
3
(xy +y
2
)

x(1/3)
0
dx
=
2
3
_
1
1/3
(x(x
1
3
) + (x
1
3
)
2
) dx
=
2
3
_
1
1/3
(2x
2
x +
1
9
) dx
=
2
3
(
2
3
x
3

1
2
x
2
+
1
9
x)

1
1/3
=
2
3
_
(
2
3

1
2
+
1
9
) (
2
81

1
18
+
1
27
)
_
= 44/243.
Thus, the probability that x < (1/3) +y is 1 (44/243) = 199/243.
1
3
x <
1
3
+ y
Figure 16.109
16.6 SOLUTIONS 1323
Problems
16. (a) We know that
_

f(x, y)dydx = 1 for a joint density function. So,


1 =
_

f(x, y)dydx =
_
1
0
_
1
x
kxydydx
=
1
8
k
hence k = 8.
(b) The region where x < y <

x is sketched in Figure 16.110


1
0
1
x
y
y = x
y =

x
Figure 16.110
So the probability that (x, y) satises x < y <

x is given by:
_
1
0
_

x
x
8xy dy dx =
_
1
0
4x(y
2
)

x
x
dx
=
_
1
0
4x(x x
2
)dx
= 4
_
1
3
x
3

1
4
x
4
_

1
0
= 4
_
1
3

1
4
_
=
1
3
This tells us that in choosing points from the region dened by 0 x y 1, that 1/3 of the time we would
pick a point from the region dened by x < y <

x. These regions are shown in Figure 16.110.


17. (a) For a density function,
1 =
_

f(x, y) dy dx =
_
2
0
_
1
0
kx
2
dy dx
=
_
2
0
kx
2
dx
=
kx
3
3

2
0
=
8k
3
.
So k = 3/8.
(b)
_
1
0
_
2y
0
3
8
x
2
dxdy =
_
1
0
1
8
(2 y)
3
dy =
1
32
(2 y)
4

1
0
=
15
32
(c)
_
1/2
0
_
1
0
3
8
x
2
dxdy =
_
1/2
0
1
8
x
3

1
0
dy =
_
1/2
0
1
8
dy =
1
16
.
1324 Chapter Sixteen /SOLUTIONS
18. Since

y
f(x, y) xy
_
R
f(x, y) dxdy
and since x never exceeds 1, and we can assume that no one lives to be over 100, so y does not exceed 100, we have
Fraction of
policies
=
_
R
f(x, y) dxdy =
_
100
65
_
1
0.8
f(x, y) dxdy,
where R is the rectangle: 0.8 x 1, 65 y 100.
19. (a) The area of S is (2)(4) = 8. Because the density function p(x, y) is constant on S and the total volume under a
density function above the xy-plane is 1, p(x, y) = 1/8 for (x, y) in S, and p(x, y) = 0 for (x, y) outside S.
(b) The probability that (x, y) is in T is
_
T
f(x, y) dy dx =
1
8
_
T
dy dx =
area(T)
8
=

8
.
20. (a) Since the exponential function is always positive and is positive, p(t) 0 for all t, and
_

0
p(t)dt = lim
b
e
t

b
0
= lim
b
e
bt
+ 1 = 1.
(b) The density function for the probability that the rst substance decays at time t and the second decays at time s is
p(t, s) = e
t
e
s
= e
ts
,
for s 0 and t 0, and is zero otherwise.
(c) We want the probability that the decay time t of the rst substance is less than or equal to the decay time s of the
second, so we want to integrate the density function over the region 0 t s. Thus, we compute
_

0
_

t
e
t
e
s
ds dt =
_

0
e
t
(e
s
)

t
dt
=
_

0
e
t
e
t
dt
=
_

0
e
(+)t
dt
=

+
e
(+)t

0
=

+
.
So for example, if = 1 and = 4, then the probability that the rst substance decays rst is 1/5 .
21. (a) If t 0, then F(t) = 0 because the average of two positive numbers can not be negative. If 1 < t then F(t) = 1
because the average of two numbers each at most 1 is certain to be less than or equal to 1. For any t, we have
F(t) =
_
R
p(x, y)dA where R is the region of the plane dened by (x + y)/2 t. Since p(x, y) = 0 outside the
unit square, we need integrate only over the part of R that lies inside the square, and since p(x, y) = 1 inside the
square, the integral equals the area of that part of the square. Thus, we can calculate the area using area formulas.
For 0 t 1, we draw the line (x + y)/2 = t, which has x- and y-intercepts of 2t. Figure 16.111 shows that for
0 < t 1/2,
F(t) = Area of triangle =
1
2
2t 2t = 2t
2
.
In Figure 16.112, when x = 1, we have y = 2t 1. Thus, the vertical side of the unshaded triangle is 1(2t 1) =
2 2t. The horizontal side is the same length, so for 1/2 < t 1,
F(t) = Area of Square Area of triangle = 1
2

1
2
(2 2t)
2
1 2(1 t)
2
.
The nal result is:
F(t) =
_
_
_
0 if t 0
2t
2
if 0 < t 1/2
1 2(1 t)
2
if 1/2 < t 1
1 if 1 < t
.
16.7 SOLUTIONS 1325
1
2t
1
2t
x+y
2
= t
x
y
Figure 16.111: For 0 < t
1
2
1 2t
2t
1
x+y
2
= t
E '
2 2t
E '
2t 1
T
c
2 2t
T
c
2t 1
x
y
Figure 16.112: For
1
2
< t 1
(b) The probability density function p(t) of z is the derivative of its cumulative distribution function. We have
p(t) =
_
_
_
0 if t 0
4t if 0 < t 1/2
4 4t if 1/2 < t 1
0 if 1 < t
.
See Figure 16.113.
(c) The values of x and y and equally likely to be near 0, 1/2, and 1. Notice from the graph of the density function in
Figure 16.113 that even though x and y separately are equally likely to be anywhere between 0 and 1, their average
z = (x +y)/2 is more likely to be near 1/2 than to be near 0 or 1.
0.5 1
2
0
x
p(t)
Figure 16.113
22. (a)
_
6
=0
_
4
r=
1
cos
p(r, )r dr d
(b)
_
6
+

12
=

6
_
4
r=
1
cos
p(r, )r dr d +
_ 2
6
=

6
+

12
_
4
r=
1
sin
p(r, )r dr d
Solutions for Section 16.7
Exercises
1. We have
(x, y)
(s, t)
=

xs xt
ys yt

5 2
3 1

= 1.
1326 Chapter Sixteen /SOLUTIONS
Therefore,

(x, y)
(s, t)

= 1.
2. We have
(x, y)
(s, t)
=

xs xt
ys yt

2s 2t
2t 2s

= 4s
2
+ 4t
2
.
Therefore,

(x, y)
(s, t)

= 4s
2
+ 4t
2
.
Notice we can drop the absolute value signs because in this case the Jacobian is nonnegative for all s and t.
3. We have
(x, y)
(s, t)
=

xs xt
ys yt

e
s
cos t e
s
sin t
e
s
sin t e
s
cos t

= (e
s
cos t)
2
+ (e
s
sin t)
2
.
Therefore,

(x, y)
(s, t)

e
2s
(cos
2
t + sin
2
t)

= e
2s
.
Notice we can drop the absolute value signs because the Jacobian in this case is positive for all s and t.
4. We have
(x, y)
(s, t)
=

xs xt
ys yt

3s
2
3t
2
6st
6st 3s
2
3t
2

= 9(s
2
t
2
)
2
+ 36s
2
t
2
.
Therefore, multiplying out and simplifying

(x, y)
(s, t)

= 9

s
4
2s
2
t
2
+t
4
+ 4s
2
t
2

= 9(s
2
+t
2
)
2
.
Notice we can drop the absolute value sign since the Jacobian in this case is nonnegative for all s and t.
5. We have
(x, y, z)
(s, t, u)
=

xs xt xu
ys yt yu
zs zt zu

3 1 2
1 5 1
2 1 1

.
This 3 3 determinant is computed the same way as for the cross product, with the entries 3, 1, 2 in the rst row playing
the same role as

i ,

j ,

k . We get
(x, y, z)
(s, t, u)
= ((5)(1) (1)(1))3 + ((1)(2) (1)(1))1 + ((1)(1) (2)(5))2 = 13.
6. We have
(x, y, z)
(r, , z)
=

xr x

xz
yr y

yz
zr z

zz

cos r sin 0
sin r cos 0
0 0 1

.
This 3 3 determinant is computed the way it is for the cross product, with the entries cos , r sin , 0 in the rst row
playing the same role as

i ,

j ,

k . We get
(x, y, z)
(s, t, u)
= (r cos 0) cos + (0 sin )(r sin ) + 0
= r cos
2
+r sin
2
= r.
7. The square T is dened by the inequalities
0 s = ax 1 0 t = by 1
that correspond to the inequalities
0 x 1/a = 10 0 y 1/b = 1
that dene R. Thus a = 1/10 and b = 1.
16.7 SOLUTIONS 1327
8. The square T is dened by the inequalities
0 s = ax 1 0 t = by 1
that correspond to the inequalities
0 x 1/a = 1 0 y 1/b = 1/4
that dene R. Thus a = 1 and b = 4.
9. The square T is dened by the inequalities
0 s = ax 1 0 t = by 1
that correspond to the inequalities
0 x 1/a = 50 0 y 1/b = 10
that dene R. Thus a = 1/50 and b = 1/10.
10. The disc S is dened by the inequality
s
2
+t
2
= a
2
x
2
+b
2
y
2
1
that corresponds to the inequality x
2
+y
2
15
2
or equivalently
1
15
2
x
2
+
1
15
2
y
2
1
that denes R. Thus a
2
= 1/15
2
and b
2
= 1/15
2
. We have a = 1/15, b = 1/15.
11. The disc S is dened by the inequality
s
2
+t
2
= a
2
x
2
+b
2
y
2
1
that corresponds to the inequality x
2
/4 + y
2
/9 1 that denes R. Thus a
2
= 1/4 and b
2
= 1/9. We have a = 1/2,
b = 1/3.
12. Inverting the change of variables gives x = s at, y = t.
The four edges of R are
y = 0, y = 3, y =
1
4
x, y =
1
4
(x 10).
The change of variables transforms the edges to
t = 0, t = 3, t =
1
4
s
1
4
at, t =
1
4
s
1
4
at
10
4
.
These are equations for the edges of a rectangle in the st-plane if the last two are of the form: s = (Constant). This
happens when the t terms drop out, or a = 4. With a = 4 the change of variables gives
_ _
T

(x, y)
(s, t)

ds dt
over the rectangle
T : 0 t 3, 0 s 10.
13. Inverting the change of variables gives x = s at, y = t.
The four edges of R are
y = 0, y = 5, y =
1
3
x, y =
1
3
(x 10).
The change of variables transforms the edges to
t = 0, t = 5, t =
1
3
s +
1
3
at, t =
1
3
s +
1
3
at +
10
3
.
These are equations for the edges of a rectangle in the st-plane if the last two are of the form: s = (Constant). This
happens when the t terms drop out, or a = 3. With a = 3 the change of variables gives
_ _
T

(x, y)
(s, t)

ds dt
over the rectangle
T : 0 t 5, 0 s 10.
1328 Chapter Sixteen /SOLUTIONS
14. Inverting the change of variables gives x = s at, y = t.
The four edges of R are
y = 15, y = 35, y = 2(x 10) + 15, y = 2(x 30) + 15.
The change of variables transforms the edges to
t = 15, t = 35, t = 2s 2at 5, t = 2s 2at 45.
These are equations for the edges of a rectangle in the st-plane if the last two are of the form: s = (Constant). This
happens when the t terms drop out, or a = 1/2. With a = 1/2 the change of variables gives
_ _
T

(x, y)
(s, t)

ds dt
over the rectangle
T : 15 t 35,
5
2
s
45
2
.
Problems
15. Given T = {(s, t) | 0 s 3, 0 t 2} and
_
x = 2s 3t
y = s 2t
The shaded area in Figure 16.114 is the corresponding region R in the xy-plane.
x
y
(0, 0)
(0, 1)
(6, 4)
(6, 3)
R
1
R
2
y =
1
2
x 1
y =
2
3
x 1
y =
2
3
x
y =
1
2
x
Figure 16.114
Since
(x, y)
(s, t)
=

x
s
x
t
y
s
y
t

2 3
1 2

= 1,

(x, y)
(s, t)

= 1.
Thus we get
_
T

(x, y)
(s, t)

ds dt =
_
3
0
ds
_
2
0
dt = 6.
Since
_
R
dxdy =
_
R
1
dxdy +
_
R
2
dxdy =
_
0
6
dx
_ 2
3
x
1
2
x1
dy +
_
6
0
dx
_ 1
2
x
2
3
x1
dy
=
_
0
6
_
1
6
x + 1
_
dx +
_
6
0
_

1
6
x + 1
_
dx = 3 + 3 = 6,
thus
_
R
dxdy =
_
T

(x, y)
(s, t)

ds dt.
16.7 SOLUTIONS 1329
16.
R
2
4
x
y
Figure 16.115
Given T = {(s, t) | 0 s 2, s t 2} and
_
x = s
2
y = t,
R = {(x, y) | 0 x 4,

x y 2}.
(x, y)
(s, t)
=

x
s
x
t
y
s
y
t

2s 0
0 1

= 2s,

(x, y)
(s, t)

= 2s since 0 s 2.
_
T

(x, y)
(s, t)

ds dt = 2
_
2
0
s ds
_
2
s
dt = 2
_
2
0
s(2 s) ds = 2
_
s
2

s
3
3
_
2
0
=
8
3
.
So,
_
R
dxdy =
_
4
0
dx
_
2

x
dy =
_
4
0
(2

x) dx
=
_
2x
2
3
x
3/2
_
4
0
= 8
16
3
=
8
3
.
Thus
_
R
dxdy =
_
T

(x, y)
(s, t)

ds dt.
17. Given
_

_
x = sin cos
y = sin sin
z = cos ,
(x, y, z)
(, , )
=

sin cos cos cos sin sin


sin sin cos sin sin cos
cos sin 0

= cos

cos cos sin sin


cos sin sin cos

+ sin

sin cos sin sin


sin sin sin cos

= cos (
2
cos
2
cos sin +
2
sin
2
cos sin )
+ sin ( sin
2
cos
2
+ sin
2
sin
2
)
=
2
cos
2
sin +
2
sin
3

=
2
sin .
1330 Chapter Sixteen /SOLUTIONS
18. Given
_
x = 3s 4t
y = 5s + 2t,
we have
_
s =
1
13
(x + 2y)
t =
1
26
(3y 5x).
Since
(x, y)
(s, t)
=

x
s
x
t
y
s
y
t

3 4
5 2

= 26,
(s, t)
(x, y)
=

s
x
s
y
t
x
t
y

1
13
2
13

5
26
3
26

=
_
3
26
__
1
13
_
+
_
5
26
__
2
13
_
=
1
26
.
So
(x, y)
(s, t)

(s, t)
(x, y)
= 26
1
26
= 1.
19. Given
_
x = 2s +t
y = s t,
we have
(x, y)
(s, t)
=

x
s
x
t
y
s
y
t

2 1
1 1

= 3,
hence

(x, y)
(s, t)

= 3.
We get
_
R
(x +y) dA =
_
T
3s

(x, y)
(s, t)

dsdt =
_
T
(3s)(3) ds dt = 9
_
T
s ds dt,
where T is the region in the st-plane corresponding to R.
x
y
(0, 0)
(3, 3)
(5, 2)
(2, 1)
R
Figure 16.116
s
t
(0, 0)
(0, 3)
(1, 0)
(1, 3)
T
Figure 16.117
Now, we need to nd T.
As
_
x = 2s +t
y = s t
or
_
s =
1
3
(x +y)
t =
1
3
(x 2y),
so from the above transformation and Figure 16.116, T is the shaded area in Figure 16.117. Therefore
_
R
(x +y) dA = 9
_
1
0
s ds
_
3
0
dt = (27)(
1
2
) = 13.5.
16.7 SOLUTIONS 1331
20. Given
_
x =
s
2
y =
t
3
,
we have
(x, y)
(s, t)
=

x
s
x
t
y
s
y
t

1
2
0
0
1
3

=
1
6
hence

(x, y)
(s, t)

=
1
6
.
So
_
R
(x
2
+y
2
) dA =
_
T
_
s
2
4
+
t
2
9
_
_
1
6
_
ds dt,
where R is the region bounded by the curve 4x
2
+9y
2
= 36 and T is the corresponding region in the st-plane. Since the
curve 4x
2
+ 9y
2
= 36 corresponds to the curve s
2
+ t
2
= 36 under the change of coordinates, T is the region bounded
by the curve s
2
+t
2
= 36. So
_
R
(x
2
+y
2
) dA =
1
24
_
T
s
2
ds dt +
1
54
_
T
t
2
ds dt.
Now polar coordinates (r, ) are used to evaluate the above integral. As
_
s = r cos
t = r sin
and
_
s
2
= r
2
cos
2

t
2
= r
2
sin
2
,

(s, t)
(r, )

cos r sin
sin r cos

= r.
So
1
24
_
T
s
2
ds dt =
1
24
_
2
0
cos
2
d
_
6
0
r
3
dr =
27
2
_
2
0
cos
2
d
=
_
27
2
__
1
2
_
_
2
0
(1 + cos 2) d =
_
27
4
_
(2) = 13.5
and
1
54
_
T
t
2
ds dt =
1
54
_
2
0
sin
2
d
_
6
0
r
3
dr = 6
_
2
0
sin
2
d = 3
_
2
0
(1 cos 2) d = (3)(2) = 6.
Thus,
_
R
(x
2
+y
2
) dA = 13.5 + 6 = 19.5.
21. The area of the ellipse is
_ _
R
dxdy where R is the region x
2
+ 2xy + 2y
2
1. We must change variables in both the
area element dA = dxdy and the region R.
Inverting the variable change gives x = s t, y = t. Thus
(x, y)
(s, t)
=

x
s
x
t
y
s
y
t

1 1
0 1

= 1.
Therefore
dxdy =

(x, y)
(s, t)

ds dt = ds dt.
The region of integration is
x
2
+ 2xy + 2y
2
= (s t)
2
+ 2(s t)t + 2t
2
= s
2
+t
2
1.
Let T be the unit disc s
2
+t
2
1. We have
_ _
R
dxdy =
_ _
T
ds dt = Area of T = .
1332 Chapter Sixteen /SOLUTIONS
22. We must change variables in the area element dA = dxdy, the integrand sin(x +y) and the region R.
Inverting the variable change gives x = (s +t)/2, y = (t s)/2. Thus
(x, y)
(s, t)
=

x
s
x
t
y
s
y
t

1/2 1/2
1/2 1/2

=
1
2
.
Therefore
dxdy =

(x, y)
(s, t)

ds dt =
1
2
ds dt.
The integrand is sin(x +y) = sin t.
The region of integration is
x
2
+y
2
=
_
s +t
2
_
2
+
_
t s
2
_
2
=
s
2
+t
2
2
1.
Let T be the disc s
2
+t
2
2 of radius

2. We have
_ _
R
sin(x +y) dxdy =
_ _
T
1
2
(sin t) ds dt = 0.
The nal integral is zero by symmetry, the integral over the part of the disc where t < 0 canceling the integral over the
part where t > 0.
23. We must change variables in the area element dA = dxdy, the integrand x and the region R.
Inverting the variable change gives x =

s t, y = s where we use the positive square root because the region R


is in the rst quadrant where x 0. Thus
(x, y)
(s, t)
=

x
s
x
t
y
s
y
t

1/(2

s t) 1/(2

s t)
1 0

=
1
2

s t
.
Therefore
dxdy =

(x, y)
(s, t)

ds dt =
1
2

s t
ds dt.
The integrand is x =

s t.
The region of integration can be transformed by examination of its boundaries. The left and right boundaries of R
are given by y x
2
= t = 0 and y x
2
= t = 9. The bottom and top boundaries of R are given by y = s = 0 and
y = s = 16.
Let T be the rectangle 0 s 16, 9 t 0 of area (9)(16) = 144. We have
_ _
R
xdxdy =
_ _
T

s t
1
2

s t
ds dt =
1
2
(Area of T) = 72.
24. We must change variables in the differential dr d, the integrand 1/r
2
and the region R.
Inverting the variable change gives r = 1/s, = t. Thus
(r, )
(s, t)
=

r
s
r
t

1/s
2
0
0 1

=
1
s
2
.
Therefore
dr d =

(r, )
(s, t)

ds dt =
1
s
2
ds dt.
The integrand is 1/r
2
= s
2
.
The region R : 1 r < , 0 2 transforms to the rectangle T : 0 s 1, 0 t 2 of area 2 in the
st-plane.
We have
_ _
R
1
r
3
r dr d =
_ _
T
s
2
1
s
2
ds dt =
_
2
0
_
1
0
ds dt = 2.
16.7 SOLUTIONS 1333
25. Given
_
s = xy
t = xy
2
,
we have
(s, t)
(x, y)
=

s
x
s
y
t
x
t
y

y x
y
2
2xy

= xy
2
= t.
Since
(s, t)
(x, y)

(x, y)
(s, t)
= 1,
(x, y)
(s, t)
= t so

(x, y)
(s, t)

=
1
t
So
_
R
xy
2
dA =
_
T
t

(x, y)
(s, t)

ds dt =
_
T
t(
1
t
) ds dt =
_
T
ds dt,
where T is the region bounded by s = 1, s = 4, t = 1, t = 4.
Then
_
R
xy
2
dA =
_
4
1
ds
_
4
1
dt = 9.
26. Let
_
s = x y
t = x +y,
that is
_
x =
1
2
(s +t)
y =
1
2
(t s),
we get
(x, y)
(s, t)
=

1
2
1
2

1
2
1
2

=
1
2
.
Hence
I =
_
R
cos
_
x y
x +y
_
dxdy =
_
T
cos
_
s
t
_

(x, y)
(s, t)

ds dt =
1
2
_
T
cos
_
s
t
_
ds dt,
where R is the triangle bounded by x + y = 1, x = 0, y = 0 and T is its image which is the triangle bounded by t = 1,
s = t, s = t.
Then
I =
1
2
_
1
0
_
t
t
cos
_
s
t
_
ds dt =
1
2
_
1
0
t[sin(1) sin(1)] dt
=
1
2
_
1
0
t 2 sin 1 dt = sin 1
_
1
0
t dt =
sin 1
2
= 0.42.
27. (a) The probability that (x + y)/2 is less than or equal to t is the integral of the joint density function p(x, y) over the
innite region (half plane) where (x +y)/2 t. Thus
F(t) =
1
2
2
_

_
2tx

e
(x
2
+y
2
)/(2
2
)
dy dx.
(b) With u = (x +y)/2, v = (x y)/2, we have x = u +v, y = u v. Thus
(x, y)
(u, v)
=

1 1
1 1

= 2
so
dxdy =

(x, y)
(u, v)

dudv = 2dudv.
1334 Chapter Sixteen /SOLUTIONS
Also x
2
+y
2
= 2(u
2
+v
2
). After writing the limits of integration in the uv-coordinates, we have
F(t) =
1
2
2
_
t

e
(u
2
+v
2
)/
2
2dv du =
2
2
2
_
t

e
u
2
/
2
_

e
v
2
/
2
dv du.
Continuing using the fact that
_

e
x
2
/a
2
dx = a

with a replaced by , we obtain


F(t) =
2
2
2
_
t

e
u
2
/
2
(

)du =
1

_
t

e
u
2
/
2
du.
(c) The probability density function of z is the derivative of its cumulative distribution function F(t). By the Fundamental
Theorem of Calculus,
p(t) = F

(t) =
1

e
t
2
/
2
.
(d) Since the density function for z can be written in the form
p(t) =
1

2(/

2)
e
t
2
/(2(/

2)
2
)
,
the distribution of z is normal, with mean 0 and standard deviation /

2.
Notice that the standard deviation of the average z = (x + y)/2 is less than the standard deviations of the
individual numbers x and y. The average of two random numbers is more likely to be near the mean than are either
of the two numbers individually.
Solutions for Chapter 16 Review
Exercises
1. We use Cartesian coordinates, oriented so that the cube is in the rst quadrant. See Figure 16.118. Then, if f is an arbitrary
function, the integral is
_
2
0
_
3
0
_
5
0
f dxdydz.
Other answers are possible. In particular, the order of integration can be changed.
2. If we imagine the disk lying horizontally, as in Figure 16.119, we can use cylindrical coordinates with the origin at the
center of the at base. Then, if f is an arbitrary function, the triple integral is
_
2
0
_
2
0
_
3
0
f r drdzd.
x
y
z
5
(5, 3, 0)
(5, 3, 2)
2
Figure 16.118
x
y
z
T
c
2 cm
'
'
3 cm
Figure 16.119
x
y
z
2
5
Figure 16.120
3. We use spherical coordinates as in Figure 16.120. Then if f is an arbitrary function, the triple integral is
_
2
0
_

/2
_
5
2
f
2
sin ddd.
Other answers are possible.
SOLUTIONS to Review Problems for Chapter Sixteen 1335
4. We use spherical coordinates, as in Figure 16.121. Then if f is an arbitrary function, the triple integral is
_
/2
0
_

0
_
5
0
f
2
sin ddd.
Other answers are possible.
5. See Figure 16.122.
x
y
z
5
5
5
0
Figure 16.121
1 1
1
1
x
y

x
2
+ y
2
= 1
Figure 16.122
2
2
x
y
E
x
2
+ y
2
= 4
Figure 16.123
6. See Figure 16.123.
7. See Figure 16.124.
y = 1
y = 4
x =

y x =

y
x
y
Figure 16.124
1

2
x
y
y = sin x or
x = sin
1
y
Figure 16.125
1
1
1
x
y
z
Figure 16.126
8. See Figure 16.125.
9. The region is the half cylinder in Figure 16.126.
10. The region of integration shown in Figure 16.127 is a three-sided pyramid bounded by the xy-plane and the planes y = 1,
z x = 0 and y x = 0.
x
y
z
(1, 1, 0)
(0, 1, 0)
(1, 1, 1)

Plane
y = x

Plane
y = 1
Figure 16.127
2 4
4
x
y
R
y = 2x + 4 or
x =
y
2
2
y = x + 4 or
x = y + 4
Figure 16.128
1336 Chapter Sixteen /SOLUTIONS
11. Integrating with respect to x rst we get
_
4
0
_
y+4
y
2
2
f(x, y) dxdy
Integrating with respect to y rst we get
_
0
2
_
2x+4
0
f(x, y) dy dx +
_
4
0
_
x+4
0
f(x, y) dy dx.
See Figure 16.128
12. Compute in polar coordinates:
_
R
_
x
2
+y
2
dA =
_

0
_
2
1
r r dr d
=
_

0
_
r
3
3
_
2
1
d
=
_

0
_
8
3

1
3
_
d =
7
3
.
13. (a) See Figure 16.129.
2
4
x
y
x = (y 4)/2 or y = 2x + 4
Figure 16.129
(b)
_
2
0
_
2x+4
0
g(x, y) dy dx.
14.
_
10
0
_
0.1
0
xe
xy
dy dx =
_
10
0
e
xy
|
0.1
0
dx
=
_
10
0
(e
0.1x
e
0
) dx
=
__
e
0.1x
0.1
_
x
_

10
0
= (10e
1
10 10e
0
)
= 10e 20 = 10(e 2).
15.
_
1
0
_
4
3
(sin(2 y)) cos (3x 7) dxdy =
_
1
0
(sin (2 y))
_
sin (3x 7)
3
_

4
3
dy
=
1
3
(sin 5 sin 2)
_
1
0
sin (2 y) dy
=
1
3
(sin 5 sin 2) [cos (2 y)]
1
0
=
1
3
(sin 5 sin 2)(cos 1 cos 2).
SOLUTIONS to Review Problems for Chapter Sixteen 1337
16.
_
1
0
_
y
0
(sin
3
x)(cos x)(cos y) dxdy =
_
1
0
(cos y)
_
sin
4
x
4
_

y
0
dy
=
1
4
_
1
0
(sin
4
y)(cos y) dy
=
sin
5
y
20

1
0
=
sin
5
1
20
.
17. First use integration by parts, with y as the variable, u = x
2
y, u

= x
2
, v =
sin (xy)
x
, v

= cos (xy). Then,


_
4
3
_
1
0
x
2
y cos (xy) dy dx =
_
4
3
_
[xy sin (xy)]
1
0

_
1
0
xsin (xy) dy
_
dx
=
_
4
3
_
xsin x + [cos (xy)]
1
0
_
dx
=
_
4
3
(xsin x + cos x 1) dx.
Now use integration by parts again, with u = x, u

= 1, v = cos x, v

= sin x. Then,
_
4
3
(xsin x + cos x 1) dx = [xcos x]
4
3
+
_
4
3
cos xdx +
_
4
3
(cos x 1) dx
= (xcos x + 2 sin x x)|
4
3
= 4 cos 4 + 2 sin 4 + 3 cos 3 2 sin 3 1.
Thus,
_
4
3
_
1
0
x
2
y cos (xy) dy dx = 4 cos 4 + 2 sin 4 + 3 cos 3 2 sin 3 1.
18.
_
1
0
_
z
0
_
2
0
(y +z)
7
dxdy dz =
_
1
0
_
z
0
2(y +z)
7
dy dz
=
_
1
0
2(y +z)
8
8

z
0
dz
=
_
1
0
(2z)
8
z
8
4
dz
=
255
4
_
1
0
z
8
dz
=
255
4
_
z
9
9
_
1
0
=
255
4

1
9
=
85
12
.
1338 Chapter Sixteen /SOLUTIONS
19. The region is shown in Figure 16.130.
1
1
0
1
x
y

1 x
2

1 x
2
Figure 16.130
The integral has the same values in the upper and lower quarter circles, so we integrate over just the upper circle and
multiply by 2. We convert the integral to polar coordinates.
_
1
0
_

1x
2

1x
2
e
(x
2
+y
2
)
dy dx = 2
_
/2
0
_
1
0
e
r
2
r dr d =
_
/2
0
(e
r
2
)

1
0
d
=
_
/2
0
1 e
1
d
=

2
(1 e
1
).
20. Evaluating gives
_
1
0
_
z
0
_
y
0
xyz dxdy dz =
_
1
0
_
z
0
x
2
yz
2

y
0
dy dz =
_
1
0
_
z
0
y
3
z
2
dy dz =
_
1
0
y
4
z
8

z
0
dz =
_
1
0
z
5
8
dz =
z
6
48

1
0
=
1
48
.
21. (a) A vertical plane perpendicular to the x-axis: x = 2.
(b) A cylinder: r = 3.
(c) A sphere: =

3.
(d) A cone: = /4.
(e) A horizontal plane: z = 5.
(f) A vertical half-plane: = /4.
22. In spherical coordinates, the spherical cap is part of the surface =

2. If is the angle at the vertex of the cone, we


have tan(/2) = 2/2 = 1, so /2 = /4. Since the cone is below the xy-plane, the angle ranges from 3/4 to .
Thus, the integral is given by
_
2
0
_

3/4
_

2
0
f(, , )
2
sin d dd.
23. In cylindrical coordinates, the spherical cap has equation z =

2 r
2
. If is the angle at the vertex of the cone, we
have tan(/2) = 2/2 = 1, so /2 = /4. The cone has equation z = r. Thus, the integral is
_
2
0
_
1
0
_
r

2r
2
g(r, , z)r dz dr d.
24. In rectangular coordinates, the spherical cap has equation z =
_
2 x
2
y
2
. If is the angle at the vertex of the
cone, we have tan(/2) = 2/2 = 1, so /2 = /4. The cone has equation z =
_
x
2
+y
2
. Thus, the integral is
_
1
1
_

1x
2

1x
2
_

x
2
+y
2

2x
2
y
2
h(x, y, z) dz dy dx.
SOLUTIONS to Review Problems for Chapter Sixteen 1339
25. From Figure 16.131, we have the following iterated integrals:
x
y
z

x
2
+ y
2
+ z
2
= 1
Figure 16.131
(a)
_
R
f dV =
_
1
1
_

1x
2

1x
2
_

1x
2
y
2
0
f(x, y, z) dzdydx
(b)
_
R
f dV =
_
1
1
_

1y
2

1y
2
_

1x
2
y
2
0
f(x, y, z) dzdxdy
(c)
_
R
f dV =
_
1
1
_

1y
2
0
_

1y
2
z
2

1y
2
z
2
f(x, y, z) dxdzdy
(d)
_
R
f dV =
_
1
1
_

1x
2
0
_

1x
2
z
2

1x
2
z
2
f(x, y, z) dydzdx
(e)
_
R
f dV =
_
1
0
_

1z
2

1z
2
_

1x
2
z
2

1x
2
z
2
f(x, y, z) dydxdz
(f)
_
R
f dV =
_
1
0
_

1z
2

1z
2
_

1y
2
z
2

1y
2
z
2
f(x, y, z) dxdydz
Problems
26. The region stands on a rectangular base in the xy-plane, with vertical sides and a slanting top, the plane z = 1 + x. See
Figure 16.132. The integral is
_
2
0
_
1
0
_
1+x
0
f(x, y, z) dz dy dx.
The order of integration can be altered.
x
y
z
1
2
c
z = 1 + x
Figure 16.132
z
z = 3
z = 2
Figure 16.133
1340 Chapter Sixteen /SOLUTIONS
27. The region is a solid ring between the planes z = 2 and z = 3, with inner radius r =

5 and outer radius r =

6. See
Figure 16.133. In cylindrical coordinates, the integral is
_
2
0
_
3
2
_

6

5
r dr dz d.
The order of integration can be changed.
28. The region stands on a circular base of radius 1 in the xy-plane and has cylindrical sides. The top is part of a sphere of
radius 3. The cylinder meets the sphere where x
2
+ y
2
= 1 and x
2
+ y
2
+ z
2
= 9, so 1 + z
2
= 9, z =

8. See
Figure 16.134. In cylindrical coordinates, the integral is
_
2
0
_
1
0
_

9r
2
0
r dz dr d.
x
z

Sphere: x
2
+ y
2
+ z
2
= 9

Cylinder: x
2
+ y
2
= 1
Figure 16.134
29. The region is a hollow half-sphere, with inner radius

3 and outer radius

4 = 2. See Figure 16.135. In spherical
coordinates, the integral is
_

0
_

0
_
2

2
sin d dd.
The order of integration can be altered.
x
y
z
c

2
Figure 16.135
30. The integral is over the region 0 x
2
+y
2
3, 1 z 4 x
2
y
2
. Using cylindrical coordinates, we get
_
2
0
_

3
0
_
4r
2
1
1
z
2
rdz dr d =
_
2
0
_

3
0
(
r
z
)

4r
2
1
dr d
SOLUTIONS to Review Problems for Chapter Sixteen 1341
=
_
2
0
_

3
0
(
r
4 r
2
+
r
1
)dr d
=
_
2
0
_
1
2
ln(4 r
2
) +
1
2
r
2
_

3
0
d
=
_
2
0
(
1
2
ln 1 +
3
2

1
2
ln 4 0)d
= 2(
3
2
ln 2) = (3 2 ln 2)
31. The region is a hemisphere 0 x
2
+y
2
+z
2
3
2
, z 0, so spherical coordinates are appropriate. Recall the conversion
formula x = sin cos . Then the integral in spherical coordinates becomes
_
2
0
_
/2
0
_
3
0
( sin cos )
2

2
sin d dd
=
_
2
0
_
/2
0
_
3
0

4
sin
3
cos
2
d dd
=
_
2
0
_
/2
0
243
5
sin
3
cos
2
dd
=
243
5
_
2
0
_
/2
0
cos
2
sin (1 cos
2
) dd
=
243
5
_
2
0
cos
2

_
cos +
1
3
cos
3

_
2
0
d
=
243
5
_
2
0
cos
2
[(1) +
1
3
(1)] d
=
243
5

2
3
_
2
0
1 + cos 2
2
d
=
81
5
( +
1
2
sin 2)

2
0
=
81
5
(2 + 0) =
162
5
32. The integral is over the region x, y 0, x
2
+y
2
1, 0 z
_
x
2
+y
2
. Using cylindrical coordinates, we get
_
/2
0
_
1
0
_
r
0
(z +r) rdz dr d =
_
/2
0
_
1
0
_
r
0
(rz +r
2
) dz dr d
=
_
/2
0
_
1
0
(
1
2
r
3
+r
3
) dr d
=
_
/2
0
3
8
r
4

1
0
d
=
3
8


2
=
3
16
33. W is a cylindrical shell, so cylindrical coordinates should be used. See Figure 16.136.
1342 Chapter Sixteen /SOLUTIONS
T
c
4
E '
1
E '
1
x
y
z
Figure 16.136
_
W
z
(x
2
+y
2
)
3/2
dV =
_
4
0
_
2
0
_
2
1
z
r
3
rdr d dz
=
_
4
0
_
2
0
_
2
1
z
r
2
dr d dz
=
_
4
0
_
2
0
(
z
r
)

2
1
d dz
=
_
4
0
_
2
0
z
2
d dz
=
_
4
0
z
2
2 dz =
1
2
z
2

4
0
= 8
34. Positive, since e
x
is always positive.
35. Negative, since y
3
is negative on B, where y < 0.
36. Positive, since (x +y
2
) is positive on R, where x > 0.
37. Cant tell, since y
3
is both positive and negative for x < 0.
38. Cant tell, since x < 0 and y
2
0 on L, where x < 0.
39. Zero. The solid sphere is symmetric and z is positive on the top half and negative (of equal absolute value) on the bottom
half. The integral of z over the entire solid is zero because the integrals over each half add up to zero.
40. Zero. x is positive on the hemisphere x
2
+y
2
+z
2
1, x > 0 and negative (of equal absolute value) on x
2
+y
2
+z
2
1,
x < 0. The integral of x over the entire solid is zero because the integrals over each half add up to zero.
41. Zero. You can see this in several ways. One way is to observe that xy is positive on the part of the sphere above and below
the rst and third quadrants (where x and y are of the same sign) and negative (of equal absolute value) on the part of the
sphere above and below the second and fourth quadrants (where x and y have opposite signs). These add up to zero in the
integral of xy over all of W.
Another way to see that the integral is zero is to write the triple integral as an iterated integral, say integrating rst
with respect to x. For xed y and z, the x-integral is over an interval symmetric about 0. The integral of x over such an
interval is zero. If any of the inner integrals in an iterated integral is zero, then the triple integral is zero.
42. Zero. Write the triple integral as an iterated integral, say integrating rst with respect to x. For xed y and z, the x-integral
is over an interval symmetric about 0. The integral of x over such an interval is zero. If any of the inner integrals in an
iterated integral is zero, then the triple integral is zero.
43. Zero. Write the triple integral as an iterated integral, say integrating rst with respect to x. Then sin(

2
xy) is integrated
over an integral symmetric about the origin, and this integral is zero because sin(

2
xy) is an odd function. Since the
innermost integral is zero so is the triple integral.
44. Positive. Since x
2
+ y
2
+ z
2
1 on the solid, we know x
2
+ y
2
1. This means that |xy| 1, since if |x| |y| then
|xy| |y
2
| = y
2
, and if |y| |x|, then |xy| |x
2
| = x
2
. So |

2
xy|

2
on the solid, and hence cos(

2
xy) is positive
on the solid W and so is its integral.
45. Negative. Since z
2
1 0 in the sphere, its integral is negative.
SOLUTIONS to Review Problems for Chapter Sixteen 1343
46. Positive. The function e
xyz
is a positive function everywhere so its integral over W is positive.
47. Positive.
_
x
2
+y
2
+z
2
is positive on W, so its integral is positive.
48. (a) The region (shaded) is one eighth of the circle x
2
+y
2
= 8; see Figure 16.137. The rst integral is above the dashed
line y = 2; the second integral is below the dashed line.
2

8
2

8
y = x
x
2
+ y
2
= 8
x
y
Figure 16.137
(b) Converting to polar coordinates, we nd the quantity in part (a) is given by
_

8
2
_

8y
2
0
e
x
2
y
2
dxdy +
_
2
0
_
y
0
e
x
2
y
2
dxdy
=
_
/2
/4
_

8
0
e
r
2
r dr d
=
_

2


4
__

1
2
e
r
2
_

8
0
=

4
_
1
2
e
0

1
2
e
8
_
=

8
_
1 e
8
_
.
49. The depth of the lake is given in meters and the diameter in kilometers. We should work with a single unit of length. In
this solution we work with kilometers, but meters would work just as well.
The shape of the lake suggests integration in polar coordinates, with r km measured from the center of the island.
Thus t = r 1 is the distance in kilometers from the island when r varies between 1 and 5. The depth of the lake r km
from the center of the island is
Depth =
100(r 1)(4 (r 1))
1000
=
1
2
+
3r
5

r
2
10
km.
Volume of the lake =
_
2
0
_
5
1
_

1
2
+
3r
5

r
2
10
_
r dr d =
32
5
= 20.1 km
3
.
50. (a) The equation of the curved surface of this half cylinder along the x-axis is (y 1)
2
+z
2
= 1. The part we want is
z =
_
1 (y 1)
2
0 y 2 0 x 10.
(b) The integral
_
D
f(x, y, z) dV =
_
10
0
_
2
0
_

1(y1)
2
0
f(x, y, z) dz dy dx.
51. Figure 16.138 shows a slice through the region for a xed x. Breaking the volume into small cubes each of volume
V = xy z and stacking the cubes above (x, y, 0), starting at z = 0 and going up to z = x +y, tells us the inner
integral is
_
x+y
0
dz.
1344 Chapter Sixteen /SOLUTIONS
Lining up the stacks parallel to the z axis gives a slice, for each xed value of x, fromy = 0 to x+y = 5, thus the middle
integral is
_
5x
0
_
x+y
0
dz dy.
Finally, adding up the contributions for x = 0 to x = 5 gives the volume, V , as
V =
_
5
0
_
5x
0
_
x+y
0
dz dy dx =
_
5
0
_
5x
0
(x +y) dy dx
=
_
5
0
(xy +
1
2
y
2
)

y=5x
y=0
dx =
_
5
0
_
x(5 x) +
1
2
(5 x)
2
_
dx =
125
3
.
z = x + y
y
z
Figure 16.138
52. (a) In cylindrical coordinates, the cone is z = r and the sphere is r
2
+z
2
= 4. The surfaces intersect where z
2
+ z
2
=
2z
2
= 4. So z =

2 and r =

2.
Volume =
_
2
0
_

2
0
_

4r
2
r
r dzdrd.
(b) In spherical coordinates, the cone is = /4 and the sphere is = 2.
Volume =
_
2
0
_
/4
0
_
2
0

2
sin ddd.
53. (a)
_
2
0
_

0
_
2
1

2
sin ddd.
(b)
_
2
0
_
2
0
_

4r
2

4r
2
r dzdrd
_
2
0
_
1
0
_

1r
2

1r
2
r dzdrd.
54. Orient the region as shown in Figure 16.139 and use cylindrical coordinates with the origin at the center of the sphere.
The equation of the sphere is x
2
+y
2
+z
2
= 25, or r
2
+z
2
= 25. If z = 3, then r
2
+ 3
2
= 25, so r
2
= 16 and r = 4.
Volume =
_
2
0
_
4
0
_

25r
2
3
r dzdrd =
_
2
0
_
4
0
rz

z=

25r
2
z=3
drd
=
_
2
0
_
4
0
(r
_
25 r
2
3r) drd =

2
0
_

(25 r
2
)
3/2
3

3r
2
2
_

4
0
= 2
__
125
3

27
3
_
24
_
=
52
3
= 54.45 cm
3
.
SOLUTIONS to Review Problems for Chapter Sixteen 1345
5
5
T
c
3
T
c
2
Sphere
r
2
+ z
2
= 25
E '
4
r
z
Figure 16.139
55. Orient the region as shown in Figure 16.140 and use spherical coordinates with the origin at the center of the sphere. The
equation of the sphere is x
2
+ y
2
+ z
2
= 25, or = 5. The plane z = 3 is the plane cos = 3, so = 3/ cos . In
Figure 16.140, angle AOB is given by
cos =
3
5
, so = arccos(3/5).
The volume is given by
V =
_
2
0
_
arccos(3/5)
0
_
5
3/ cos

2
sin ddd =

2
0
_
arccos(3/5)
0
sin

3
3

p=5
p=3/ cos
d
= 2
_
arccos(3/5)
0
_
125
3

9
cos
3

_
sin d
= 2
__
arccos(3/5)
0
125
3
sin d
_
arccos(3/5)
0
9
cos
3

sin d
_
= 2
_
_

125
3
cos
_

arccos(3/5)
0
9
_
1
2 cos
2

arccos(3/5)
0
_
= 2
_

125
3
_
3
5
1
_

9
2
_
1
(3/5)
2
1
__
= 2
_

125
3
_

2
5
_

9
2
_
16
9
__
= 2
_
50
3
8
_
=
52
3
54.45 cm
3
.
O

Sphere
= 5
B
A
T
c
3
T
c
2
'
'
5
x
z
Figure 16.140
56. The density function can be rewritten as (, , ) = . So the mass is
_
W
(P) dV =
_
2
0
_
/4
0
_
3
0

2
sin d dd
=
_
2
0
_
/4
0
81
4
sin dd
1346 Chapter Sixteen /SOLUTIONS
=
81
4
_
2
0
(

2
2
+ 1) d
=
81
4
2 (

2
2
+ 1) =
81
4
(

2 + 2)
57. The region of integration is shown in Figure 16.141, and the mass of the given solid is given by
x
y
z
3
4
12
z = 12 4x 3y
'
4x + 3y = 12
or y =
1
3
(12 4x)
Figure 16.141
Mass =
_
R
dV
=
_
3
0
_ 1
3
(124x)
0
_
124x3y
0
x
2
dzdydx
=
_
3
0
_ 1
3
(124x)
0
x
2
z

z=124x3y
z=0
dydx
=
_
3
0
_ 1
3
(124x)
0
x
2
(12 4x 3y) dydx
=
_
3
0
x
2
(12y 4xy
3
2
y
2
)

y=
1
3
(124x)
0
dx
=
_
8x
3
4x
4
+
8
15
x
5
_

3
0
=
108
5
.
58. We use spherical coordinates because we are integrating over a sphere and the density has spherical symmetry. D = 2.
M =
_
2
0
_

0
_
3
0
(2)
2
sin d dd.
SOLUTIONS to Review Problems for Chapter Sixteen 1347
59. Let the lower left part of the forest be at (0, 0). Then the other corners have coordinates as shown. The population density
function is then given by
(x, y) = 10 2y
The equations of the two diagonal lines are x = 2y/5 and x = 6 + 2y/5. So the total rabbit population in the forest is
_
5
0
_
6+
2
5
y

2
5
y
(10 2y) dxdy =
_
5
0
(10 2y)(6 +
4
5
y) dy
=
_
5
0
(60 4y
8
5
y
2
) dy
= (60y 2y
2

8
15
y
3
)

5
0
= 300 50
8
15
125
=
2750
15
=
550
3
183
60. We must rst decide on coordinates. We pick cylindrical coordinates with the z-axis along the axis of the cylinders. The
insulation stretches from z = 0 to z = l. See Figure 16.142. The volume is given by the integral
Volume =
_
2
0
_
l
0
_
a+h
a
r drdzd.
Evaluating the integral gives
Volume =
_
2
0
_
l
0
r
2
2

a+h
a
dzd = 2z

l
0
_
(a +h)
2
2

a
2
2
_
= l((a +h)
2
a
2
).
To check our answer, notice that the volume is the difference between the volume of two cylinders of radius a and a +h.
These cylinders have volumes l(a +h)
2
and la
2
, respectively.
x
y
z
a
a + h
E '
h
E '
a
T
c
l
Figure 16.142
61. The plane (x/p) + (y/q) + (z/r) = 1 cuts the axes at the points (p, 0, 0); (0, q, 0); (0, 0, r). Since p, q, r are positive,
the region between this plane and the coordinate planes is a pyramid in the rst octant. Solving for z gives
z = r
_
1
x
p

y
q
_
= r
rx
p

ry
q
.
The volume, V , is given by the double integral
V =
_
R
_
r
rx
p

ry
q
_
dA,
1348 Chapter Sixteen /SOLUTIONS
where R is the region shown in Figure 16.143. Thus
V =
_
p
0
_
qqx/p
0
_
r
rx
p

ry
q
_
dydx
=
_
p
0
_
_
ry
rxy
p

ry
2
2q
_

y=qqx/p
y=0
_
dx
=
_
p
0
_
r
_
q
qx
p
_

r
p
x
_
q
qx
p
_

r
2q
_
q
qx
p
_
2
_
dx
=
_
p
0
rq
2rqx
p
+
rqx
2
p
2

rq
2
2q
_
1
2x
p
+
x
2
p
2
_
dx
=
_
rqx
rqx
2
p
+
rqx
3
p
2
3

rqx
2
+
rqx
2
p2

rqx
3
2p
2
3
_

p
0
= pqr pqr +
pqr
3

pqr
2
+
pqr
2

pqr
6
=
pqr
6
.
p
q
R
_
_
_
x
p
+
y
q
= 1
y = q
qx
p

x
y
Figure 16.143
62. We must rst decide on coordinates. We imagine the vertex of the cone downward, at the origin, with the at base in the
plane z = h, as in Figure 16.144. Then, using cylindrical coordinates as in Figure 16.145, we see that the curved surface
of the cone has equation z = hr/a. Thus the volume is given by
Volume =
_
2
0
_
a
0
_
h
hr/a
r dzdrd.
Evaluating gives
Volume =
_
2
0
_
a
0
rz

z=h
z=hr/a
drd =
_
2
0
_
a
0
_
hr
hr
2
a
_
drd
= 2
_
hr
2
2

hr
3
3a
_

a
0
= 2h
_
a
2
2

a
2
3
_
=
ha
2
3
.
x
y
z
E '
a
T
c
h
Figure 16.144
a
h
z = hr/a
r
z
Figure 16.145
SOLUTIONS to Review Problems for Chapter Sixteen 1349
63. We use spherical coordinates. Since the density, , is equal to the distance from the point to the origin, we have
= gm/cm
3
.
Therefore the mass of the hemisphere is given by
Mass =
_
2
0
_
/2
0
_
2
0

2
sin ddd =
_
2
0
_
/2
0

4
4

2
0
sin dd
= 4
_
2
0
_
/2
0
sin dd = 4
_
2
0
(cos )

/2
0
d = 4 2 1 = 8 gm.
64. Since the hole resembles a cylinder, we will use cylindrical coordinates. Let the center of the sphere be at the origin, and
let the center of the hole be the z-axis (see Figure 16.146).
x
y
z
a

a
2
R
2
Figure 16.146
Then we will integrate from z =

a
2
R
2
to z =

a
2
R
2
, and each cross-section will be an annulus. So the
volume is
_

a
2
R
2

a
2
R
2
_
2
0
_

a
2
z
2
R
r dr d dz =
_

a
2
R
2

a
2
R
2
_
2
0
1
2
(a
2
z
2
R
2
) d dz
=
_

a
2
R
2

a
2
R
2
(a
2
z
2
R
2
) dz
=
_
(a
2
R
2
)(2
_
a
2
R
2
)
1
3
(2(a
2
R
2
)
3
2
)
_
=
4
3
(a
2
R
2
)
3
2
65. We must rst decide on coordinates. We pick Cartesian coordinates with the smaller sphere centered at the origin, the
larger one centered at (0, 0, 1). A vertical cross-section of the region in the xz-plane is shown in Figure 16.147. The
smaller sphere has equation x
2
+y
2
+z
2
= 1. The larger sphere has equation x
2
+y
2
+ (z + 1)
2
= 2.
1350 Chapter Sixteen /SOLUTIONS
1

x
2
+ y
2
+ z
2
= 1

x
2
+ y
2
+ (z + 1)
2
= 2
x
z
Figure 16.147
Let R represent the region in the xy-plane which lies directly underneath (or above) the region whose volume we
want. The curve bounding this region is a circle, and we nd its equation by solving the system:
x
2
+y
2
+z
2
= 1
x
2
+y
2
+ (z + 1)
2
= 2
Subtracting the equations gives
(z + 1)
2
z
2
= 1
2z + 1 = 1
z = 0.
Since z = 0, the two surfaces intersect in the xy-plane in the circle x
2
+y
2
= 1. Thus R is x
2
+y
2
1.
The top half of the small sphere is represented by z =
_
1 x
2
y
2
; the top half of the large sphere is represented
by z = 1 +
_
2 x
2
y
2
. Thus the volume is given by
Volume =
_
1
1
_

1x
2

1x
2
_

1x
2
y
2
1+

2x
2
y
2
dzdydx.
Starting to evaluate the integral, we get
Volume =
_
1
1
_

1x
2

1x
2
(
_
1 x
2
y
2
+ 1
_
2 x
2
y
2
) dydx.
We simplify the integral by converting to polar coordinates
Volume =
_
2
0
_
1
0
_
_
1 r
2
+ 1
_
2 r
2
_
r drd
=
_
2
0
_

(1 r
2
)
3/2
3
+
r
2
2
+
(2 r
2
)
3/2
3
_

1
0
d
= 2
_
1
2
+
1
3

_

1
3
+
2
3/2
3
__
= 2
_
7
6

2

2
3
_
= 1.41.
66. Suppose the brick is set up as shown in Figure 16.148.
x
y
z
T
c
1
'
'
'
'
5
3
Figure 16.148
SOLUTIONS to Review Problems for Chapter Sixteen 1351
The brick has m/v = density = 1. The moment of inertia about the z-axis is
Iz =
_
5/2
5/2
_
3/2
3/2
_
1/2
1/2
1(x
2
+y
2
) dz dy dx
=
_
5/2
5/2
_
3/2
3/2
(x
2
+y
2
) dy dx
=
_
5/2
5/2
(3x
2
+
9
4
) dx
=
125
4
+
45
4
=
85
2
The moment of inertia about the y-axis is
Iy =
_
5/2
5/2
_
3/2
3/2
_
1/2
1/2
1(x
2
+z
2
) dz dy dx
=
_
5/2
5/2
_
3/2
3/2
(x
2
+
1
12
) dy dx
=
_
5/2
5/2
(3x
2
+
1
4
) dx
=
125
4
+
5
4
=
65
2
The moment of inertia about the x-axis is
Ix =
_
5/2
5/2
_
3/2
3/2
_
1/2
1/2
1(y
2
+z
2
) dz dy dx
=
_
5/2
5/2
_
3/2
3/2
(y
2
+
1
12
) dy dx
=
_
5/2
5/2
(
9
4
+
1
4
) dx
= 5
10
4
=
25
2
67. Let the ball be centered at the origin. Since a ball looks the same from all directions, we can choose the axis of rotation;
in this case, let it be the z-axis. It is best to use spherical coordinates, so then
x
2
+y
2
= ( sin cos )
2
+ ( sin sin )
2
=
2
sin
2

Then m/v = Density = 1, so the moment of inertia is


Iz =
_
R
0
_
2
0
_

0
1(
2
sin
2
)
2
sin dd d
=
_
R
0
_
2
0
_

0

4
(sin )(1 cos
2
) dd d
=
_
R
0
_
2
0

4
(cos +
1
3
cos
3
)

0
d d
=
_
R
0
_
2
0
4
3

4
d d
=
_
R
0
8
3

4
d =
8
15
R
5
1352 Chapter Sixteen /SOLUTIONS
68. Set up the cylinder with the base centered at the origin on the xy plane, facing up. (See Figure 16.149.) Newtons Law of
Gravitation states that the force exerted between two particles is
F = G
m1m2

2
where G is the gravitational constant, m1 and m2 are the masses, and is the distance between the particles. We take a
small volume element, so m1 = m, and m2 = dV . In cylindrical coordinates, if m is at (0,0,0) and dV is at (r, , z),
(see Figure 16.149), then the distance from m to dV is given by =

r
2
+z
2
for r1 r r2 and 0 z h.
z
T
c
z
r
m

'
dV
E '
r
1
E '
r
2
Figure 16.149
Due to the symmetry of the cylinder the sum of all the horizontal forces is zero; the net force on m is vertical. The force
acting on the particle as a result of the small piece dV makes an angle with the vertical and therefore has vertical
component
Vertical force on
particle from small
piece of cylinder
=
GmdV
(

r
2
+z
2
)
2
cos =
GmdV
r
2
+z
2

z

r
2
+z
2
=
Gmz
(r
2
+z
2
)
3
2
dV.
Thus, since dV = rdzdrd,
Total force =
_
2
0
_
r
2
r
1
_
h
0
Gmz
(r
2
+z
2
)
3/2
r dzdrd
= 2Gm
_
r
2
r
1
_
h
0
zr
(r
2
+z
2
)
3/2
drdz
= 2Gm
_
r
2
r
1
_
1
r
(r
2
+h
2
)
1
2
_
dr
= 2Gm(r (r
2
+h
2
)
1
2
)

r
2
r
1
= 2Gm(r2 r1
_
r
2
2
+h
2
+
_
r
2
1
+h
2
).
69. The outer circle is a semicircle of radius 4. This is shown in Figure 16.150, with center at D. Thus, CE = 2 and DC = 2,
while AD = 4. Notice that angle ADO is a right angle.
SOLUTIONS to Review Problems for Chapter Sixteen 1353
O
r 2
D
2
C
2
E
A
4
r
B
Figure 16.150
Suppose the large circle has center O and radius r. Then OA = r and OD = OC DC = r 2. Applying
Pythagoras Theorem to triangle OAD gives
r
2
= 4
2
+ (r 2)
2
r
2
= 16 +r
2
4r + 4
r = 5.
If we put the origin at O, the equation of the large circle is x
2
+ y
2
= 25. In the same coordinates, the equation of the
small circle, which has center at D = (3, 0), is (x 3)
2
+y
2
= 16. The right hand side of the two circles are given by
x =
_
25 y
2
and x = 3 +
_
16 y
2
.
Since the y-coordinate of A is 4 and the y-coordinate of B is 4, we have
Area =
_
4
4
_
3+

16y
2

25y
2
1 dxdy
=
_
4
4
(3 +
_
16 y
2

_
25 y
2
) dy
= 13.95.
70. Lets denote the (x, y) coordinates of the points in the lagoon by L. Since x and y are measured in kilometers and d is
measured in meters, and 1 km = 1000 m, the volume of a small piece of the lagoon is given by
V d(x, y)(1000x)(1000y)m
3
.
Thus, the total volume of the lagoon is given by
V = 1000
2
_
L
d(x, y) dxdy.
Changing coordinates using u = x/2 and v = y f(x) converts the depth function to:
d(x(u, v), y(u, v)) = 40 160v
2
160u
2
= 160(
1
4
u
2
v
2
) meters.
Thus, the points in the lagoon have (u, v) coordinates in the disk, D, given by u
2
+v
2
1/4.
The Jacobian of the transformation is:

x
u
x
v
y
u
y
v

2 0
2f

(2u) 1

= 2.
Thus, the integral in u, v coordinates is
V = 1000
2
_
L
d(x, y) dxdy = 10
6
_
D
160(
1
4
u
2
v
2
)2 dudv = 320 10
6
_
D
(
1
4
u
2
v
2
) dudv.
Converting to polar coordinates, we have
V = 320 10
6
_
2
0
_
1/2
0
(
1
4
r
2
)r drd = 320 10
6
2(
1
4
r
2
2

r
4
4
)

1/2
0
= 10
7
m
3
.
1354 Chapter Sixteen /SOLUTIONS
71. (a) The constant k is determined by the condition that
_
R
k(x + y)dA = 1 where the region R is the quarter disk with
radius 10
x
2
+y
2
100 x 0 y 0.
Using polar coordinates gives the integral
_
R
k(x +y)dA =
_
/2
0
_
10
0
k(r cos +r sin )r dr d
= k
_
/2
0
_
10
0
r
2
(cos + sin )dr d
= k
_
/2
0
1000
3
(cos + sin )d
=
1000k
3
(sin cos )

2
0
=
1000k
3
2 =
2000k
3
.
Since 2000k/3 = 1, we have k = 3/2000.
(b) Evaluate the integral
_
S
fdA where S is the region 0 r 7, 0 /2. We have
_
S
fdA =
3
2000
_
/2
0
_
7
0
r
2
(cos + sin )dr d =
3
2000
_
/2
0
7
3
3
(cos + sin )d
=
7
3
2000
(sin cos )

/2
0
=
7
3
2000
2 =
343
1000
.
The probability that the point is closer than 7 units from the origin is 343/1000.
CAS Challenge Problems
72. The region is the triangle to the right of the y-axis, below the line y = 1, and above the line y = x. Thus the integral can
be written as
_
1
0
_
1
x
e
y
2
dydx or as
_
1
0
_
y
0
e
y
2
dxdy. The second of these integrals can be evaluated easily by hand:
_
1
0
_
y
0
e
y
2
dxdy =
_
1
0
_
e
y
2
x

x=y
x=0
_
dy =
_
1
0
ye
y
2
dy
=
1
2
e
y
2

1
0
=
1
2
(e 1)
The other integral cannot be done by hand with the methods you have learned, but some computer algebra systems will
compute it and give the same answer.
73. In Cartesian coordinates the integral is
_
D
3
_
x
2
+y
2
dA =
_
1
1
_

1x
2

1x
2
3
_
x
2
+y
2
dydx.
In polar coordinates it is
_
D
3
_
x
2
+y
2
dA =
_
2
0
_
1
0
3

r
2
rdrd =
_
2
0
_
1
0
r
5/3
drd
=
_
2
0
3
8
d =
6
7
The Cartesian coordinate version requires the use of a computer algebra system. Some CASs may be able to handle it
and may give the answer in terms of functions called hypergeometric functions. To compare the answers are the same you
may need to ask the CAS to give a numerical value for the answer. Its possible your CAS will not be able to handle the
integral at all.
74.
_
1
0
_
0
1
x +y
(x y)
3
dydx = 1/2 and
_
0
1
_
1
0
x+y
(xy)
3
dxdy = 1/2. This does not contradict the theorem because the
function is not continuous everywhere inside the region of integration. In fact, it is not even dened at the origin.
CHECK YOUR UNDERSTANDING 1355
75.
Average value for F =
1
Area
_
h
h
_
h
h
(a +bx
4
+cy
4
+dx
2
y
2
+ex
3
y
3
) dxdy
=
1
4h
2
_
4ah
2
+
4bh
6
5
+
4ch
6
5
+
4dh
6
9
_
= a +
1
45
(9b + 9c + 5d) h
4
The limit is
lim
h0
(a +
1
45
(9b + 9c + 5d) h
4
) = a.
Notice that F(0, 0) = a.
Average value for G =
1
4h
2
_
h
h
_
h
h
(a sin(kx) +b cos(ky) +c) dxdy
=
1
4h
2
_
4
_
ch
2
k +bhsin(hk)
_
k
_
= c +
b sin(hk)
hk
.
The limit is
lim
h0
_
c +
b sin(hk)
hk
_
= c +b lim
h0
sin(hk)
hk
= b +c.
Notice that G(0, 0) = b +c.
Finally,
Average value for H =
(a +b)
_
1 +e
2h
_ _
2 2h h
2
+e
2h
_
2 2h +h
2
__
4e
2h
h
2
.
You may need to simplify the answer given by your CAS to get this form. The limit of this as h 0 (calculated with a
CAS) is 0. This is equal to H(0, 0).
In each case the limit of the average values over smaller and smaller squares centered at the origin is equal to the value
of the function at the origin. We conjecture that this is true in general for a continuous function. This makes sense because
when the square is small, the function is approximately constant on the square with value equal to its value at the origin.
Therefore the integral is approximately the area times the value of the function, so the average value is approximately the
value of the function. This approximation gets better and better as h 0.
CHECK YOUR UNDERSTANDING
1. False. For example, if f(x, y) < 0 for all (x, y) in the region R, then
_
R
f dA is negative.
2. True. The double integral is the limit of the sum

f(x, y)A =

kA = k

A
over rectangles that lie inside the region R. As the area A 0, this sum approaches k Area(R).
3. False. The function f(x, y) = e
xy
is largest at the (1, 1) corner of R, so for any (x, y) in R we have e
xy
e
11
= e.
Then
_
R
e
xy
dA = lim
A0

e
xy
A lim
A0

eA = e lim
A0

A = e Area(R) = e.
So
_
R
e
xy
dA e 2.7.
4. False. For example, if f = 1, then
_
R
1 dA = Area(R) = 6 and
_
S
1 dA = Area(S) = 6.
1356 Chapter Sixteen /SOLUTIONS
5. True. The double integral is the limit of the sum

A0
(x, y)A. Each of the terms (x, y)A is an approximation of
the total population inside a small rectangle of area A. Thus the limit of the sum of all of these numbers as A
gives the total population of the region R.
6. False. If the graph of f has equal volumes above and below the xy-plane over the region R, the double integral is zero
without having f(x, y) = 0 everywhere.
7. True. Writing the denition of the integral of g, we have
_
R
g dA = lim
A0

g(x, y)A = lim


A0

kf(x, y)A = k lim


A0

f(x, y)A = k
_
R
f dA.
8. False. As a counterexample, let R be a rectangle with area 2 and take f(x, y) = g(x, y) = 1. Then
_
R
f g dA =
_
R
1 dA = Area(R) = 2, but
_
R
f dA =
_
R
g dA = Area(R) Area(R) = 4.
9. False. There is no reason to expect this to be true, since the behavior of f on one half of R can be completely unrelated
to the behavior of f on the other half. As a counterexample, suppose that f is dened so that f(x, y) = 0 for points
(x, y) lying in S, and f(x, y) = 1 for points (x, y) lying in the part of R that is not in S. Then
_
S
f dA = 0, since
f = 0 on all of S. To evaluate
_
R
f dA, note that f = 1 on the square S1 which is 0 x 1, 1 y 2. Then
_
R
f dA =
_
S
1
f dA = Area(S1) = 1, since f = 0 on S.
10. True. Since all points in the region Rsatisfy x < y, it is true that at every point in R, f(x, y) = x+x < x+y = g(x, y).
Since all of the values of f in R are less than those of g, the average of the values of f is less than the average of the
values of g.
11. False. Since the inside integral is performed with respect to x and the outside integral with respect to y, the region of
integration is the rectangle 5 x 12, 0 y 1.
12. False. The iterated integral
_
2
0
_
1
0
f dxdy is over a rectangle. The correct limits are
_
1
0
_
2x
x
f dydx.
13. True. For any point in the region of integration we have 1 x 2, and so y is between the positive numbers 1 and 8.
14. False. The sign of
_
b
a
_
d
c
fdydx depends on the behavior of the function f on the region of integration. For example,
_
2
1
_
2
1
(x)dydx =
3
2
.
15. False. The integrals are over different regions, so there is no reason to expect their values to be equal. The region of
integration of
_
1
0
_
x
0
f dydx is the triangle with vertices (0, 0), (1, 1) and (1, 0), while the region of
_
1
0
_
y
0
f dxdy is the
triangle with vertices (0, 0), (1, 1) and (0, 1).
16. True. Since f does not depend on x, the inside integral (which is with respect to x) evaluates to
_
1
0
fdx = xf

x=1
x=0
=
(f 0) = f. Thus
_
b
a
_
1
0
f dxdy =
_
b
a
f dy.
17. False. The given limits describe only the upper half disk where y 0. The correct limits are
_
a
a
_

a
2
x
2

a
2
x
2
fdydx.
18. True. As an example, take the region inside the triangle with vertices (0, 0), (1, 1) and (2, 3). Iterated integration over this
region using either the dxdy or the dydx orders requires breaking the region into two pieces:
_
1
0
_
3x/2
x
f dydx +
_
2
1
_
3x/2
2x1
f dydx or
_
1
0
_
y
2y/3
f dxdy +
_
3
1
_ 1
2
(y+1)
2y/3
f dxdy
19. True. In the inner integral with respect to y, the function g(x) can be treated as a constant, so
_
b
a
_
d
c
g(x)h(y) dxdy =
_
b
a
g(x)
__
d
c
h(y) dy
_
dx.
The result of the integral
_
d
c
h(y) dy is a constant, so may be factored out of the integral with respect to x. Thus we have
_
b
a
g(x)
__
d
c
h(y) dy
_
dx =
__
d
c
h(y) dy
_

__
b
a
g(x) dx
_
.
PROJECTS FOR CHAPTER SIXTEEN 1357
20. False. As a counterexample, consider
_
2
0
_
2
0
(x +y) dxdy. We have
_
2
0
_
2
0
(x +y) dxdy =
_
2
0
_
x
2
2
+yx
_

x=2
x=0
dy =
_
2
0
(2 + 2y) dy = 8
and
_
2
0
xdx +
_
2
0
y dy =
x
2
2

x=2
x=0
+
y
2
2

y=2
y=0
= 2 + 2 = 4.
21. False. The integral gives the total mass of the material contained in W.
22. True. The region lies above the square 0 x 1, 0 y 1 and below the plane z = x.
23. False. The given limits only cover the part of the unit ball in the rst octant where x 0, y 0, and z 0. To cover the
entire unit ball the limits are
_
1
1
_

1x
2

1x
2
_

1x
2
y
2

1x
2
y
2
f dzdydx.
24. True. Both sets of limits describe the solid region lying above the triangle x +y 1, x 0, y 0, z = 0 and below the
plane x +y +z = 1.
25. True. Both sets of limits describe the solid region lying above the rectangle 1 x 1, 0 y 1, z = 0 and below
the parabolic cylinder z = 1 x
2
.
26. False. The iterated integral is of the form like
_
b
a
_
d
c
_
k
e
fdz dy dx only if the rectangular region has faces parallel to the
coordinate axes. More general rectangular regions, such as a cube with one corner at the origin and the opposite corner at
(0, 0, 1) will need to be written as the sum of iterated integrals where the limits are not constant.
27. False. As a counterexample, consider f(x, y, z) =
1
2
x. Then f is positive on half the cube and negative on the other
half. Symmetry can be used to show that
_
1
0
_
1
0
_
1
0
(
1
2
x)dz dy dx = 0.
28. True. Since
_
W
f dV = lim

i,j,k
f(xi, yj, z
k
)V, where (xi, yj, z
k
) is a point inside the ijk-th sub-box of volume V,
and since f > g, we have
lim

i,j,k
f(xi, yj, z
k
)V > lim

i,j,k
g(xi, yj, z
k
)V =
_
W
g dV.
29. False. As a counterexample, let W1 be the solid cube 0 x 1, 0 y 1, 0 z 1, and let W2 be the solid cube

1
2
x 0,
1
2
y 0,
1
2
z 0. Then volume(W1) = 1 and volume(W2) =
1
8
. Now if f(x, y, z) = 1, then
_
W
1
f dV = 1 1 which is less than
_
W
2
f dV =
1
8
1.
30. True. If W is the solid region lying under the graph of f and above the region R in the xy-plane, we can compute the
volume of W either using the double integral
_
R
f dA, or using the triple integral
_
W
1 dV.
PROJECTS FOR CHAPTER SIXTEEN
1. (a) We are integrating over the whole plane, so converting to polar coordinates gives
_

e
(x
2
+y
2
)
dxdy =
_
2
0
_

0
e
r
2
rdrd =
_
2
0

1
2
e
r
2

0
d =
_
2
0
1
2
d = .
(b) Rewriting the integrand as a product gives
_

e
(x
2
+y
2
)
dxdy =
_

e
x
2
e
y
2
dxdy.
Now e
y
2
is a constant as far as the integral with respect to x is concerned, so
_

e
x
2
e
y
2
dxdy =
_

e
y
2
__

e
x
2
dx
_
dy.
1358 Chapter Sixteen /SOLUTIONS
We assume that the integral with respect to x converges, and so is a constant as far as the integral with
respect to y is concerned. Thus, we have
_

e
y
2
__

e
x
2
dx
_
dy =
__

e
x
2
dx
___

e
y
2
dy
_
.
But
_

e
x
2
dx and
_

e
y
2
dy are the same number, so we can write
_

e
(x
2
+y
2
)
dxdy =
__

e
x
2
dx
___

e
y
2
dy
_
=
__

e
x
2
dx
_
2
.
(c) Using the results of parts (a) and (b), we have
__

e
x
2
dx
_
2
=
_

e
(x
2
+y
2
)
dxdy = .
Taking square roots and observing that the integral we are looking for is positive, we have
_

e
x
2
dx =

.
2. (a) We want to nd the average value of |x y| over the square 0 x 1, 0 y 1:
Average distance between gates =
_
1
0
_
1
0
|x y| dy dx.
Lets x x, with 0 x 1. Then |x y| =
_
y x for y x
x y for y x
. Therefore
_
1
0
|x y| dy =
_
x
0
(x y) dy +
_
1
x
(y x) dy
=
_
xy
y
2
2
_

x
0
+
_
y
2
2
xy
_

1
x
= x
2

x
2
2
+
1
2
x
x
2
2
+ x
2
= x
2
x +
1
2
.
So,
Average distance between gates =
_
1
0
_
1
0
|x y| dy dx
=
_
1
0
__
1
0
|x y| dy
_
dx =
_
1
0
(x
2
x +
1
2
) dx
=
x
3
3

x
2
2
+
1
2
x

1
0
=
1
3
.
(b) There are (n + 1)
2
possible pairs (i, j) of gates, i = 0, . . . , n, j = 0, . . . , n, so the sum given represents
the average distances apart of all such gates. The Riemann sum with x = y = 1/n, if we choose the
least x and y-values in each subdivision is
n1

i=0
n1

j=0

i
n

j
n

1
n
2
,
which for large n is just about the same as the other sum. For n = 5 the sum is about 0.389; for n = 10
the sum is about 0.364.

You might also like